Vous êtes sur la page 1sur 68

TEST - 22

User Name :

chandan paswan

Total Marks :

200

Mark Scored :

76

TS

IA

1 Why tank irrigation is more practiced in South India as compared to other forms of
irrigation like canal and wells?
1. It is difficult to dig canals and wells due to undulating relief and hard rock structure.
2. Low percolation of rain water due to hard rock structure
3. Many rivers of South India are seasonal in nature.
4. Population and agricultural fields are scattered which makes canal irrigation
economically unviable.
Select the correct answer using the codes below.
A. 1, 2 and 3 only
B. 1 and 4 only
C. 2 and 3 only
D. 1, 2, 3 and 4
User Answer : D
Correct Answer : D
Answer Justification :
Justification:Tank is an artificial reservoir. Tank use is critical in parts of South
India without perennial rainfall where water supply replenishment is dependent on
monsoon seasons.

IN
SI

Most of the rivers of this region are seasonal and dry up in summer season.
Therefore, they cannot supply water to canals throughout the year.
There are several streams which become torrential during rainy season. The
only way to make best use of this water is to impound it by constructing
bunds and building tanks. Otherwise this water would go waste to the sea.
Most of the tanks are natural and do not involve heavy cost for their
construction. Even an individual farmer can have his own tank.
Tanks are generally constructed on rocky bed and have longer life span. In
many tanks, fishing is also carried on. This supplements both the food
resources and income of the farmer.

Q Source: Improvisation: UPSC CDS 2015

2 The system of price controls was an important component of Alauddin Khalji's market
reforms. Why were price controls placed?
A. It was the duty of a welfare state.
B. To reduce the remuneration paid to the large army
C. Gain more competitiveness in grain trade with neighbouring states
D. Stabilizing markets from unnecessary price inflation by producers
User Answer : C

(C) Insights Active Learning. | All rights reserved.

www.insightsias.com

TEST - 22

User Name :

chandan paswan

Total Marks

200

Mark Scored

76

Correct Answer : B
Answer Justification :
Justification & Learning:Khilji's military ambitions required a standing and strong
army, especially after the Mongol siege of Delhi.

IA

Maintaining a large army at regular salaries, however, would be severe drain on the
treasury. A system of price controls reduced the salary amount that needed to be
paid.
He placed markets under the control of a high officer called 'Shahna' for strictly
controlling the shopkeepers and prices.
Land revenue was fixed and the grain was stored in government granaries.

TS

Q Source: Improvisation: UPSC CDS 2015

IN
SI

3 The Mahar movement and Shri Narayana Dharma Paripalana Yogam (SNDP) Movement
were two important backward class movements in India. What was the ideological
difference between these two movements?
A. Former relied on total rejection of Hinduism, but the latter relied at a new
interpretation of Hinduism for reform.
B. Former followed non-violence as a core ideology, whereas the latter believed
in violent dismantling of the government apparatus.
C. Former believed in ground work for caste reforms, whereas the latter believed
in joining Central and Provincial legislatures to achieve caste reforms by
political means
D. All of the above
User Answer :
Correct Answer : A
Answer Justification :
Justification & Learning:When Ambedkar lead the Mahar movement, he rejected
the Gandhian approach relying on change of the heart of caste Hindu and of
incorporating the Harijan in the Sudra Varna; he decided to abandon Hinduism
altogether and along with five lakhs Mahars embraced Buddhism.
SNDP had a different approach.
Narayana Guru (founder of SNDP) built a number of temples, which were declared
open to all castes.

(C) Insights Active Learning. | All rights reserved.

www.insightsias.com

TEST - 22

User Name :

chandan paswan

Total Marks :

200

Mark Scored :

76

He also simplified rituals regarding marriage, religious worship, and funerals.


Narayana Guru achieved a notable success in transforming the untouchable groups
into a backward class.

He criticized Gandhi for his faith in Chaturvarna, which he considered the parent of
the caste system and untouchability. He gave a new slogan "one religion, one caste
and one God for mankind".

IA

Q Source:Lower caste movements in Modern India: Modern History

TS

4 Consider the following statements.Assertion (A): Highest concentration of plankton is


found at tropics and subtropics.Reason (R): Sunlight can be found abundantly at tropics
and sub-tropics that is vital for the growth of plankton.In the context of the above, which of
these is correct?
A. A is correct, and R is an appropriate explanation of A.
B. A is correct, but R is not an appropriate explanation of A.
C. A is correct, but R is incorrect.
D. A is correct, but R is incorrect.
User Answer : A
Correct Answer : D
Answer Justification :

Justification:Highest concentration of plankton is found at higher latitudes, tropics


and subtropics have lower concentrations.

IN
SI

It is due to nutrient limitation brought about by strong, year-round thermocline


(temperature gradients) and pycnocline (salinity gradients).
Pycnocline refers to the rapid change in density with depth, and thermocline, a rapid
change in temperature with depth. Both of these limit circulation of nutrients to the
surface where planktons grow.
However, in areas where ocean upwelling occurs, nutrients are cycled to the surface
and rich plankton grounds can be found.
Q Source: Concepts of Ecology

5 Areas near the equator receive more direct solar radiation than areas near the poles.
However, these areas do not constantly get warmer and warmer. Why?
1. Regions near equator reflect all the heat received back to space.

(C) Insights Active Learning. | All rights reserved.

www.insightsias.com

TEST - 22

User Name :

chandan paswan

Total Marks

200

Mark Scored

76

IA

2. Ocean waves and winds transport the heat from the lower latitudes to higher
latitudes.
Which of the above is/are correct?
A. 1 only
B. 2 only
C. Both 1 and 2
D. None
User Answer : B
Correct Answer : D
Answer Justification :

Justification: Statement 1: The earth as a whole reflects all the heat, but the same
cannot be said of a particular place or region, for e.g. equator. So, 1 is incorrect.

TS

Statement 2: It is not waves that transport heat, it is Ocean currents. Waves do not
move water but only the energy. It is the fundamentals of physics.

Large quantities of heat can be absorbed and stored in the surface layers of the
ocean. This heat is transported by ocean currents. In this way, the ocean currents
help regulate Earth's climate by facilitating the transfer of heat from warm tropical
areas to colder areas near the poles.

Q Source:11th NCERT: Geography

IN
SI

6 The Financial Sector Legislative Reforms Commission (FSLRC) has recommended


revamping the legislative framework governing the financial sector by a non-sectoral,
principle-based approach. Which of the following is/are recommendations of FSLRC?
1. The entire financial sector should be governed by a unified agency.
2. There is a need for enhancing transparency in the functioning of existing financial
sector regulators.
3. Government will manage its public debt by itself and not be dependent on an outside
agency.
Select the correct answer using the codes below.
A. 1 only
B. 2 and 3 only
C. 2 only
D. 1, 2 and 3
User Answer :
Correct Answer : C
Answer Justification :

(C) Insights Active Learning. | All rights reserved.

www.insightsias.com

TEST - 22

User Name :

chandan paswan

Total Marks :

200

Mark Scored :

76

Justification: Statement 1 and 3: The FSLRC recommended a seven agency


structure for the financial sector which are the Reserve Bank of India (RBI), Unified
Financial Agency (UFA), Financial Sector Appellate Tribunal (FSAT), Resolution
Corporation (RC), Financial Redressal Agency (FRA), Financial Stability and
Development Council (FSDC) and Public Debt Management Agency (PDMA).

IA

Statement 2: The non-legislative aspects of the recommendations are relating to


governance enhancing measures on consumer protection and greater transparency in
the functioning of financial sector regulators.
The legislative recommendations relate to re-writing the laws using a principle
based approach, restructuring existing regulatory agencies and creating new
agencies.

TS

Q Source: Current affairs of last year (2015) http://pib.nic.in/newsite/PrintRelease.aspx?relid=108381

IN
SI

7 What was/were the essential change(s) brought by the 73rd constitutional amendment
Act in the Panchayati Raj system in India?
1. Panchayats were created by Act as they did not exist in Independent India.
2. Panchayati Raj System received constitutional status.
3. It fixed the size of Panchayats and granted executive power to the panchs.
Select the correct answer using the codes below.
A. 1 and 2 only
B. 2 only
C. 1 and 3 only
D. 1, 2 and 3
User Answer : B
Correct Answer : B
Answer Justification :
Justification:Statement 1: Rajasthan was the first state to establish Panchayati Raj
in 1959. Other states also followed suit, so 1 is wrong.
Statement 2: Though most of the states created panchayati raj institutions by mid
1960s, there were differences from one state to another with regard to the number of
tiers, relative position of samiti and parishad, their tenure, composition, functions,
finances and so on.
The 73rd Act harmonized all the systems into a common structure and granted PRIs
constitutional status.

(C) Insights Active Learning. | All rights reserved.

www.insightsias.com

TEST - 22

User Name :

chandan paswan

Total Marks

200

Mark Scored

76

Statement 3: Panchayat is elected from area wards, and its size is not fixed by the
constitution.
Q Source:Indian Polity: M Laxmikanth

TS

IA

8 A group of officers known as Dhamma mahamattas were instituted in the Ashokan


empire to
1. Implement and publicize the various aspects of Dhamma
2. Design religious codes that should be followed by people in general
Which of the above is/are correct?
A. 1 only
B. 2 only
C. Both 1 and 2
D. None
User Answer : A
Correct Answer : A
Answer Justification :

Justification: Ashoka attacked ceremonies and sacrifices as meaningless. A group


of officers known as Dhamma mahamattas were instituted to implement and
publicize the various aspects of Dhamma.

IN
SI

Ashoka made them responsible for carrying his message to various sections of
society, However, they gradually developed into a type of priesthood of Dhamma
with great powers and soon began to interfere in politics.
The aspects of Dhamma were developed chronologically.
Q Source: 11th TamilNadu Textbook

9 Rosetta space probe built by the European Space Agency (ESA) landed on a comet
millions of miles ahead of earth. How will Rosetta help the scientific community?
A. It will help in understanding the fundamental design of the Universe.
B. Since comets revolve around the Sun, it will help us know more about the
inner core of Sun.
C. We will understand the nature of fundamental particles since the space probe
is shielded from cosmic rays.
D. None of the above
User Answer :
Correct Answer : D

(C) Insights Active Learning. | All rights reserved.

www.insightsias.com

TEST - 22

User Name :

chandan paswan

Total Marks :

200

Mark Scored :

76

Answer Justification :
Learning: Previous observations have shown that comets contain complex organic
compounds like amino acids. Such compounds are essential ingredients for life.

Comets are thought to have delivered a vast quantity of water to Earth, and they may
have also seeded Earth with organic molecules.

IA

Rosetta and Philae will also search for organic molecules, nucleic acids (the building
blocks of DNA and RNA) and amino acids (the building blocks of proteins) by
sampling and analysing the comet's nucleus and coma cloud of gas and dust, helping
assess the contribution comets made to the beginnings of life on Earth.

TS

Q
Source:http://www.thehindu.com/sci-tech/science/rosetta-probe-makes-key-discove
ry/article7278294.ece

IN
SI

10 The model of demographic transition helps us understand the relationship between


A. Social change and patterns of population growth
B. Historical patterns of development between fertile and non-fertile regions.
C. Patterns of population growth and economic development
D. History of migration and geographical reasons for the same
User Answer : C
Correct Answer : C
Answer Justification :
Learning: Demographic transition (DT) refers to the transition from high birth and
death rates to lower birth and death rates as a country develops from a pre-industrial
to an industrialized economic system.
This can be divided into four stages:
Birth Rate and Death rate are both high. Population growth is slow and
fluctuating, e.g. LDCs.
Birth Rate remains high. Death Rate is falling. Population begins to rise
steadily, e.g. Nigeria.
Birth Rate starts to fall. Death Rate continues to fall. Population rising, e.g.
China, Brazil.
Birth Rate and Death Rate both low. Population steady.Typical of USA;
Sweden; Japan; Britain

(C) Insights Active Learning. | All rights reserved.

www.insightsias.com

TEST - 22

User Name :

chandan paswan

Total Marks

200

Mark Scored

76

Q Source:11th NCERT: Fundamentals of Human Geography

TS

IA

11 Volcanoes are active in which of the following islands in or around India?


1. Andaman and Nicobar Islands
2. Lakshadweep
3. Kachatheevu Islands
Select the correct answer using the codes below.
A. 1 only
B. 2 and 3 only
C. 2 only
D. 1 and 2 only
User Answer : A
Correct Answer : A
Answer Justification :

Justification: Statement 1: Barren Island is located in the Andaman Sea, one of the
most easterly islands in the Andaman Islands chain. It is the only confirmed active
volcano in South Asia. There were eruptions in 1994-95 and 2005-07. The waters
surrounding Barren Island are reputed to be among the world's top scuba diving
destinations.

Narcondum and Bartatang are other volcanoes in Andaman.


Dinodhar Hill is an inactive volcano near Kutch, Gujarat.

IN
SI

Dhosi Hill is an extinct volcano in the North-West end of the Aravali mountain
range.
Other places in statement 2 and 3 do not host volcanoes.
Q Source:9th NCERT: Geography

12 This beach separates the Bhitarkanika mangroves from the Bay of Bengal and is the
world's most important nesting beach for Olive Ridley Sea Turtles. The beach is also a part
of a Marine Wildlife Sanctuary. It is
A. Calangute
B. Gopalpur-on-Sea
C. Mandarmani
D. Gahirmatha
User Answer :

(C) Insights Active Learning. | All rights reserved.

www.insightsias.com

TEST - 22

User Name :

chandan paswan

Total Marks :

200

Mark Scored :

76

Correct Answer : D
Answer Justification :
Learning:Odisha coast has the world's largest known rookery of Olive Ridley sea
turtle.

TS

IA

Apart from Gahirmatha rookery, two other mass nesting beaches have been
located which are on the mouth of rivers Rushikulya and Devi.
The spectacular site of mass congregation of Olive Ridley sea turtles for
mating and nesting enthralls both the scientists and the nature lovers
throughout the world.
This unique phenomenon is hardly seen anywhere in India. That rarity of
mass congregation and the cruelty meted out to these innocent marine
creatures, paved the way for declaration of the stretch of water body from old
light house near Batighar to Maipura river mouth as Gahirmatha (Marine)
Wildlife Sanctuary.
Gahirmatha is the only marine wildlife sanctuary of Odisha.

Q Source:Improvisation: UPSC IES 2014

IN
SI

13 India has many endemic plant and vertebrate species and endemism rich areas. Which
one among the following regions comes last in the richness of endemic population?
A. Eastern Himalayas
B. Andaman and Nicobar Islands
C. Gangetic plains
D. Western Ghats
User Answer :
Correct Answer : C
Answer Justification :
Justification & Learning: Among plants, species endemism is estimated at 33%.
Areas rich in endemism are north-east India, the Western Ghats and the northwestern and eastern Himalayas.
A small pocket of local endemism also occurs in the Eastern Ghats.
The Gangetic plains are generally poor in endemics, while the Andaman and
Nicobar Islands contribute at least 220 species to the endemic flora of India.
Endemism among mammals and birds is relatively low. In contrast, endemism in the

(C) Insights Active Learning. | All rights reserved.

www.insightsias.com

TEST - 22

User Name :

chandan paswan

Total Marks

200

Mark Scored

76

Indian reptilian and amphibian fauna is high.

Q Source: Improvisation: 10th NCERT Geography

Only 55 bird species are endemic to India, with distributions concentrated in areas
of high rainfall. They are located mainly in eastern India along the mountain chains
where the monsoon shadow occurs, south-west India (the Western Ghats), and the
Nicobar and Andaman Islands.

TS

IA

14 Indian National Army (INA) was a force formed by Indian nationalists to secure Indian
independence from British rule. Which of the following events coincide with the formation
of INA?
A. Non-cooperation movement
B. Civil Disobedience Movement
C. First World War
D. Second World War
User Answer : D
Correct Answer : D
Answer Justification :

Learning:It was formed in 1942 in Southeast Asia during World War II under
Mohan Singh, by Indian PoWs of the British-Indian Army captured by Japan in the
Malayan campaign and at Singapore.

IN
SI

It was revived under the leadership of Subhas Chandra Bose after his arrival in
Southeast Asia in 1943.
A number of people associated with the INA during the war later went on to hold
important roles in public life in India as well as in other countries in Southeast Asia,
most notably Lakshmi Sehgal in India.
Q Source:UPSC past year papers

15 Post Independence, the first major development programme launched in India was
Community Development Programme, 1952. This programme was formulated to provide
A. Certain sums of revenue to each village in the country for creation of physical
and human capital
B. An administrative framework through which the government might reach to
the district, block and village level
C. Grass-root planning structure for informal local bodies

(C) Insights Active Learning. | All rights reserved.

www.insightsias.com

10

TEST - 22

User Name :

chandan paswan

Total Marks :

200

Mark Scored :

76

D. All of the above


User Answer :
Correct Answer : B
Answer Justification :
Learning: All the districts of the country were divided into "Development Blocks".

TS

IA

A "Block Development Officer (BDO)" was made in charge of each block.


Below the BDO, Village Level Workers (VLW) were appointed who were
responsible to keep in touch with some villages. So, a nationwide structure
was started to be created.
BDOs and VLW's were trained for the job of carrying out array of
government programmes and make it possible to reach the government to
villages.
Top authority was "Community Development Organization" and a
Community Development Research Center was created with best academic
brains of the country at that time.

Q Source:Chapter 24: India Yearbook 2016 (UPSC had asked about an old scheme
last year too)

IN
SI

16 Consider the following about the Deen Dayal Upadhyaya Grameen Kaushalya Yojana
which is a restructured version of the Ajeevika Mission - NRLM.
1. It provides demand led skill training at no cost to the rural poor.
2. Target Group is identified through Special Development Officers (SDOs) especially
appointed for this purpose.
3. It does not cover the North-eastern states and J&K.
Select the correct answer using the codes below.
A. 1 and 2 only
B. 2 and 3 only
C. 1 only
D. 1, 2 and 3
User Answer : C
Correct Answer : C
Answer Justification :
Learning:Features of Deen Dayal Upadhyaya Grameen Kaushalya Yojana:
Enable Poor and Marginalized to Access Benefits
Demand led skill training at no cost to the rural poor
Inclusive Program Design

(C) Insights Active Learning. | All rights reserved.

www.insightsias.com

11

TEST - 22

User Name :

chandan paswan

Total Marks

200

Mark Scored

76

You can read more at Q source.

IA

Mandatory coverage of socially disadvantaged groups (SC/ST 50%;


Minority 15%; Women 33%)
Shifting Emphasis from Training to Career Progression
Pioneers in providing incentives for job retention, career progression
and foreign placements
Greater Support for Placed Candidates
Post-placement support, migration support and alumni network
Proactive Approach to Build Placement Partnerships
Guaranteed Placement for at least 75% trained candidates

Q Source: http://pib.nic.in/newsite/efeatures.aspx?relid=115288

TS

Chapter 24: India Yearbook 2016

IN
SI

17 Winters are very cold in the continental steppes of Eurasia when compared to the steppe
type climate in Southern hemisphere due to
A. The large geographical extent of steppes in Eurasia
B. Blowing of Chinook winds in Southern hemisphere
C. Presence of warm Humboldt Current in the Southern hemisphere
D. Large distance from sea in Eurasia
User Answer : B
Correct Answer : D
Answer Justification :
Learning: We can take Pretoria in South Africa as an example, which is located
near Velds grasslands.
It has a subtropical climate, which accounts for its short winters, which are relatively
dry, and its long but wet summers.
But, since it is located near the sea, temperature is moderate heavily.
However, in Eurasia, due to enormous distance from sea, the moderating effects of
the sea do not affect the climate at all, and the winters become very cold.
Chinook blows in the Northern hemisphere. So, (b) is incorrect.
Q Source:Chapter 20: Goh Cheng Leong: Certificate Physical and Human
Geography

(C) Insights Active Learning. | All rights reserved.

www.insightsias.com

12

TEST - 22

User Name :

chandan paswan

Total Marks :

200

Mark Scored :

76

IA

18 The focus of National Urban Transport Policy 2014 formulated by the Central
Government is
A. "Move People Not Vehicles"
B. "One city One transport"
C. "Cut Emissions Not number of vehicles"
D. "One city One transport Tariff"
User Answer :
Correct Answer : A
Answer Justification :

Learning:The vision statement of the National Urban Transport Policy (NUTP)


acknowledges that people occupy centre-stage, and that cities must evolve in a
manner that supports the main social and economic activities taking place there.

TS

The NUTP aims to bring about equitable use of road space with people and not
vehicles as the focus.

The NUTP is more of a policy framework containing general guidelines rather than
rules that are implemented. Further, transport is a state subject, thus making the
issue of access to the Central government a tricky issue.

Considering this, the Centre had to tie include some components of NUTP in
JNNURM, and make the funding dependent on the implementation. This is how
many cities have introduced BRTS (Bus Rapid Transit System). This has also led
states to explore setting up of metros.

IN
SI

Q Source: Chapter 24: India Yearbook 2016

19 Consider the following statements about money bills with regard to State legislature.
1. A money bill can be introduced in the legislative assembly only and that too on the
recommendation of the governor.
2. Since every such bill is considered to be a government bill, it can be introduced only
by a minister.
Which of the above is/are correct?
A. 1 only
B. 2 only
C. Both 1 and 2
D. None
User Answer : A
Correct Answer : C
Answer Justification :

(C) Insights Active Learning. | All rights reserved.

www.insightsias.com

13

TEST - 22

User Name :

chandan paswan

Total Marks

200

Mark Scored

76

Learning: A Money Bill cannot be introduced in the legislative council.


After a Money Bill is passed by the legislative assembly, it is transmitted to the
legislative council for its consideration. The legislative council has restricted powers
with regard to a Money Bill.

Q Source: Indian Polity: M Laxmikanth

IA

It cannot reject or amend a Money Bill. It can only make recommendations and must
return the bill to the legislative assembly within 14 days. The legislative assembly
can either accept or reject all or any of the recommendations of the legislative
council.

IN
SI

TS

20 There has been a lot of focus in India on developing Cryogenic Engines indigenously.
What advantage does cryogenic engine has over conventionally used rocket engines?
A. It does not require fuel and can operate by burning atmospheric gases as fuel.
B. It is more efficient than provides greater thrust for the same amount of
propellant burnt.
C. The engine does not have any emissions when compared to a conventional
engine.
D. Only cryogenic engines can take us to the geostationary orbits, conventional
engines have not been able to.
User Answer :
Correct Answer : B
Answer Justification :
Learning: A Cryogenic rocket stage is more efficient and provides more thrust for
every kilogram of propellant it burns compared to solid and earth-storable liquid
propellant rocket stages. Specific impulse (a measure of the efficiency) achievable
with cryogenic propellants (liquid Hydrogen and liquid Oxygen) is much higher
compared to earth storable liquid and solid propellants, giving it a substantial
payload advantage.
However, cryogenic stage is technically a very complex system compared to solid or
earth-storable liquid propellant stages due to its use of propellants at extremely low
temperatures and the associated thermal and structural problems.
Q Source:Chapter 25: India Yearbook 2016
21 The Speaker of the Lok Sabha derives his powers and duties from

(C) Insights Active Learning. | All rights reserved.

www.insightsias.com

14

TEST - 22

User Name :

chandan paswan

Total Marks :

200

Mark Scored :

76

A. Constitution of India
B. Rules of Procedure and Conduct of Business of Lok Sabha
C. Parliamentary Conventions which are unwritten rules
D. All of the above
User Answer : A
Correct Answer : D
Answer Justification :

IA

Learning:Option (a): For e.g. his power to decide whether a bill is money bill or
not.
Option (b): His powers to regulate the business or the house and set the house in
order etc.

TS

Option (c): Rules like Mavalankar rule where the Speaker recognizes the leader of
the largest opposition party as the Leader of opposition.
Q Source: Indian Polity: M Laxmikanth

IN
SI

22 Endeavour of the ASTROSAT mission launched by Indian Space and Research


Organization (ISRO) is to
A. Provide better agricultural forecasts for improving crop yields
B. Give us a more accurate estimate of impending climate changes on earth
C. Understand the Universe in more detail
D. Plan positions for future satellite orbits so as to shield them from large
astronomical objects
User Answer :
Correct Answer : C
Answer Justification :
Learning: ASTROSAT is India's first dedicated multi wavelength space
observatory. This scientific satellite mission endeavours for a more detailed
understanding of our universe.
ASTROSAT will observe universe in the optical, Ultraviolet, low and high energy
X-ray regions of the electromagnetic spectrum, whereas most other scientific
satellites are capable of observing a narrow range of wavelength band.
The scientific objectives of ASTROSAT mission are:
To understand high energy processes in binary star systems containing

(C) Insights Active Learning. | All rights reserved.

www.insightsias.com

15

TEST - 22

User Name :

chandan paswan

Total Marks

200

Mark Scored

76

neutron stars and black holes


Estimate magnetic fields of neutron stars
Study star birth regions and high energy processes in star systems lying
beyond our galaxy
Detect new briefly bright X-ray sources in the sky
Perform a limited deep field survey of the Universe in the Ultraviolet region
Q Source:Current Affairs & Improvisation: Chapter 25: India Yearbook 2016

TS

IA

23 Which of the following can be the longest lasting greenhouse gas in the atmosphere?
A. Fluorinated gases
B. Methane
C. Nitrous oxide
D. Argon
User Answer : C
Correct Answer : A
Answer Justification :

Learning: Methane, by contrast, is mostly removed from the atmosphere by


chemical reaction, persisting for about 12 years. Thus although methane is a potent
greenhouse gas, its effect is relatively short-lived.

Nitrous oxide is destroyed in the stratosphere and removed from the atmosphere
more slowly than methane, persisting for around 114 years.

IN
SI

Fluorinated gases are removed from the atmosphere only when they are destroyed
by sunlight in the far upper atmosphere. In general, fluorinated gases are the most
potent and longest lasting type of greenhouse gases emitted by human activities.
The lifetime in the air of CO2, the most significant man-made greenhouse gas, is
probably the most difficult to determine, because there are several processes that
remove carbon dioxide from the atmosphere.
Q Source: Improvisation: ICSE Environmental Studies Xth

24 A steady increase in the concentration of Greenhouse Gases (GHGs) in Earth's


atmosphere can affect India in which of the following ways?
1. Accelerated melting of glaciers with intensification of monsoon can lead to flood
disasters in the Himalayan catchments.
2. Sea level can rise at the Indian coasts.

(C) Insights Active Learning. | All rights reserved.

www.insightsias.com

16

chandan paswan

Total Marks :

200

Mark Scored :

76

IA

3. Increased temperatures can affect agricultural production.


Select the correct answer using the codes below.
A. 1 and 2 only
B. 2 only
C. 1 and 3 only
D. 1, 2 and 3
User Answer : D
Correct Answer : D
Answer Justification :

TEST - 22

User Name :

TS

Justification & Learning: Statement 1: It is projected that by the end of the 21st
century, rainfall will increase by 15 - 31% and high concentration of GHGs will
increase the mean annual temperature by about 3 degree Celsius. This may lead to
melting of glacier and the increased rainfall to floods in the catchment areas.
Statement 2: Water expands on heating and contribution of melting of glaciers will
cause sea level rise. It can have the following effects:

Deltas will be threatened by flooding, erosion and salt intrusion;


Loss of coastal mangroves will have impact on fisheries
Effect on livelihoods and local communities

Statement 3: Plants grow in a narrow range of temperature. Increase in average


temperature also results in more pests and insects attack, and an overall decline in
agricultural productivity and production.

IN
SI

Q Source:Ecology concepts

25 A Biosphere Reserve seeks to conserve


A. Only biological diversity
B. Both biological and cultural diversity
C. Biological diversity, cultural diversity and entire landscape of the reserve
D. Only certain species that may be critically endangered
User Answer : C
Correct Answer : C
Answer Justification :
Learning:Biosphere Reserves (BRs) are representative parts of natural and cultural
landscapes extending over large area of terrestrial or coastal/marine ecosystems or a
combination thereof and representative examples of bio-geographic
zones/provinces.

(C) Insights Active Learning. | All rights reserved.

www.insightsias.com

17

TEST - 22

User Name :

chandan paswan

Total Marks

200

Mark Scored

76

The UNESCO has introduced the designation 'Biosphere Reserve' for natural areas
to minimize conflict between development and conservation.
Areas potential for preservation of traditional tribal or rural modes of living for
harmonious use of environment are also covered in Biosphere reserves. So, (c) is the
most appropriate option.

Q Source: ICSE Environmental Studies Xth

TS

IA

26 Consider the following statements.


1. The Mughal School of painting originated in the reign of Akbar.
2. The Western Indian style of painting is the only style uninfluenced by infusion from
foreign style.
Which of the above is/are correct?
A. 1 only
B. 2 only
C. Both 1 and 2
D. None
User Answer : A
Correct Answer : A
Answer Justification :

IN
SI

Justification: Statement 1: Akbar's reign (1556-1605) ushered a new era in Indian


miniature painting. After he had consolidated his political power, he built a new
capital at Fatehpur Sikri where he collected artists from India and Persia.
Statement 2: During the 15th century the Persian style of painting started
influencing the Western Indian style of painting as is evident from the Persian facial
types and hunting scenes appearing on the borders of some of the illustrated
manuscripts of the Kalpasutra.
Q Source:http://www.ccrtindia.gov.in/visualarts.php

27 How is the Chairman of Legislative Council selected?


A. Nominated by the Governor from amongst the members of the Council
B. Selected by the Speaker of the Legislative Assembly
C. Elected by the council itself from amongst its members.
D. The Senior-most member of the Council automatically becomes its
Chairman.
User Answer : C

(C) Insights Active Learning. | All rights reserved.

www.insightsias.com

18

TEST - 22

User Name :

chandan paswan

Total Marks :

200

Mark Scored :

76

Correct Answer : C
Answer Justification :
Learning: As a presiding officer, the powers and functions of the Chairman in the
council are similar to those of the Speaker in the assembly.

IA

However, the Speaker has one special power which is not enjoyed by the Chairman.
The Speaker decides whether a bill is a Money Bill or not and his decision on this
question is final.
As in the case of the Speaker, the salaries and allowances of the Chairman are also
fixed by the state legislature. They are charged on the Consolidated Fund of the
State and thus are not subject to the annual vote of the state legislature.

TS

He resigns by writing to the Deputy Chairman. He can be removed by a resolution


passed by a majority of all the then members of the council.
Q Source: Indian Polity: M Laxmikanth

IN
SI

28 In the Mauryan Empire, what was the duty of Pradeshikas and Rajukas?
A. They worked in judicial capacity.
B. They were responsible for maintaining land records for purpose of tax
collection.
C. They managed royal treasury.
D. They presided over local body meetings.
User Answer : D
Correct Answer : A
Answer Justification :
Learning:Kautilya refers to the existence of two kinds of courts - dharmasthiyas
(dealing with civil matters) and kantakasodhanas (dealing criminal cases).
There were special courts in the cities and villages presided over by the pradesika,
mahamatras and rajukas.
The Pradesika were also the principal police officers, whose duty was to investigate
the crimes com?mitted in the region within their jurisdiction.
Rajukas were next in hierchy to Pradesikas.
Q Source: 11th Tamil Nadu Textbook

(C) Insights Active Learning. | All rights reserved.

www.insightsias.com

19

TEST - 22

User Name :

chandan paswan

Total Marks

200

Mark Scored

76

IA

29 Consider the following about Shola forests.


1. It is an evergreen ecosystem found only at high altitude regions in tropics.
2. It can be found in Kerala and Tamilnadu.
3. Sholas are generally found at sites where adjacent slopes converge.
Select the correct answer using the codes below.
A. 1 and 2 only
B. 2 and 3 only
C. 1 and 3 only
D. 1, 2 and 3
User Answer : C
Correct Answer : D
Answer Justification :

TS

Learning: The Sholas are a mosaic of mountane evergreen forests and grasslands.
They are found only in high altitude (>1500 metres asl) regions within the tropics,
and are limited to the southern part of the Western Ghats.

They are characterised by undulating grassland patches, interspersed with thickets of


stunted evergreen tree species, and are home to a host of endemic and endangered
plants and animals.

They are also vitally important in keeping water cycles alive. They retain most of
the rain they get over the monsoons, and release it slowly through the year via a
network of streams and rivers that eventually serve the needs of a huge number of
human settlements across south India.

IN
SI

Q Source:NCERT 10th Geography

30 Consider the following about Bengal School of Painting that flourished throughout
India during the British Raj in the early 20th century.
1. They followed modern western art forms and dismissed the ancient Indian art as
primitive.
2. It only painted themes related to Indian freedom struggle so as to generate
nationalistic consciousness.
3. It was promoted and supported by British who were arts administrators in India.
Select the correct answer using the codes below.
A. 1 and 2 only
B. 2 and 3 only
C. 3 only
D. None of the above

(C) Insights Active Learning. | All rights reserved.

www.insightsias.com

20

TEST - 22

User Name :

chandan paswan

Total Marks :

200

Mark Scored :

76

User Answer : D
Correct Answer : C
Answer Justification :

Justification: This new group of painters rejected the art of Raja Ravi Varma as
imitative and westernized. They declared that such a style was unsuitable for
depicting the nation's ancient myths and legends.

IA

Statement 1 and 2: The turned to the inspiration to medieval Indian traditions of the
miniature paintings and ancient art of mural paintings in Ajanta Caves. The
paintings of Ajanta and Bagh, Mogul, Rajput and Pahari miniatures provided the
models.

TS

The continuity of earlier traditions was sought to be maintained by borrowing from


legends and classical literature like the Ramayana, the Mahabharata, Gita, and
Puranas, the writings of Kalidasa and Omar Khayyam.

Statement 3: Also known as 'Indian style of painting' in its early days, it was
associated with Indian nationalism and led by Rabindaranath Tagore, but was also
promoted and supported by British arts administrators like E. B. Havell, the
principal of the Government College of Art, Kolkata from 1896; eventually it led to
the development of the modern Indian painting.

Q Source:http://www.ccrtindia.gov.in/modernindianpainting.php

IN
SI

31 The President can NOT act without the advice of the Council of Ministers in which of
the following cases?
A. Appointment of Prime Minister when no party has a clear majority in the Lok
Sabha
B. Dismissal of the council of ministers when it cannot prove the confidence of
the Lok Sabha
C. Appointment of Governors in States
D. Dissolution of the Lok Sabha if the council of ministers has lost its majority
User Answer : C
Correct Answer : C
Answer Justification :
Learning:Though the President has no constitutional discretion, he has some
situational discretion.
In other words, the President can act on his discretion (that is, without the advice of

(C) Insights Active Learning. | All rights reserved.

www.insightsias.com

21

TEST - 22

User Name :

chandan paswan

Total Marks

200

Mark Scored

76

the ministers) under the following situations:


When the Prime Minister in office dies suddenly and there is no obvious
successor.
Or when it would be wrong for the Council of Ministers to advice the
President, for e.g. after losing majority in Lok Sabha.

Q Source: Indian Polity: M Laxmikanth

TS

IA

32 In modern texts on Indian Buddhist architecture, the term chaitya-griha is often used to
denote
A. A prayer hall housing a stupa
B. A dwelling place for monks
C. Core of the Buddhist monasteries
D. Open space outside the monasteries used for public functions
User Answer : A
Correct Answer : A
Answer Justification :

Learning: A chaitya is a Buddhist shrine or prayer hall with a stupa at one end.

Chaityas were probably constructed to hold large numbers of devotees and to


provide shelter for them.

IN
SI

Architecturally, chaityas show similarities to ancient Roman architectural concepts


of column and arch.
About 1200 such cave temples were built throughout India. The most important of
these are the Karla Caves, Ajanta Caves, Ellora Caves, Udayagiri and Khandagiri
Caves, Aurangabad Caves and the Pandavleni Caves.
Q Source:http://www.ccrtindia.gov.in/buddhist_architecture.php

33 The Government of India or any of the State Governments have launched dedicated
projects (like Project Tiger) for the conservation of all of these species except
A. Elephant
B. Dolphin
C. Hangul
D. Crocodile
User Answer : C

(C) Insights Active Learning. | All rights reserved.

www.insightsias.com

22

TEST - 22

User Name :

chandan paswan

Total Marks :

200

Mark Scored :

76

Correct Answer : B
Answer Justification :
Learning: Project Elephant (PE), a centrally sponsored scheme, was launched in
February 1992 to provide financial and technical support to major elephant bearing
States in the country for protection of elephants, their habitats and corridors

IA

The Kashmir stag also called Hangul is a subspecies of Central Asian Red Deer
native to northern India.
The state of Jammu & Kashmir, along with the IUCN and the WWF prepared a
project for the protection of these animals. It became known as Project Hangul.

TS

The Indian Crocodile Conservation Project is considered among the more successful
of conservation initiatives in the world.
Only a conservation plan for Dolphins has been launched. So, (b) is the correct
answer.

You can more about the status of Dolphin (India's national aquatic animal) here
http://www.bbc.com/news/science-environment-19255703

Q Source:ICSE Environmental Studies Xth

IN
SI

34 The basic cause of Monsoon climate is the


A. Difference in the rate of heating and cooling of land and sea
B. Upper air circulation along with jet streams
C. Alternation of warm and cold currents
D. Presence of obstructing mountains in the path of planetary winds
User Answer : A
Correct Answer : A
Answer Justification :
Learning: The monsoon is essentially the seasonal reversal in wind direction.
The apparent position of the Sun with reference to the Earth oscillates from
the Tropic of Cancer to the Tropic of Capricorn. Thus, the low pressure
regions created by solar heating also changes latitude.
The northeast and southeast trade winds converge in this low pressure zone,
which is also known as the Intertropical Convergence Zone or ITCZ.
This low pressure regions sees continuous rise of moist wind from the sea

(C) Insights Active Learning. | All rights reserved.

www.insightsias.com

23

TEST - 22

User Name :

chandan paswan

Total Marks

200

Mark Scored

76

surface to the upper layers of the atmosphere, where the cooling means the air
can no longer hold so much moisture resulting in precipitation.
The rainy seasons of East Asia, sub-Saharan Africa, Australia and the
southern parts of North America coincide with the shift of ITCZ towards
these regions.

Q Source: Chapter 16: Goh Cheng Leong: Certificate Physical and Human
Geography

TS

IA

35 The eastern coasts of continents within the tropics have much heavier rainfall than the
interiors or the west coasts. This is because
1. All western coasts fall in the rain shadow zone.
2. Trade winds cause precipitation blowing to the eastern side.
Which of the above is/are correct?
A. 1 only
B. 2 only
C. Both 1 and 2
D. None
User Answer : B
Correct Answer : B
Answer Justification :

IN
SI

Justification: Islands and coastal areas 10 to 20 north or south of the equator


usually have a tropical marine climate. The ocean is the main influence in creating
the tropical marine climate. The temperature ranges from 25C to 35C. The trade
winds blow all year round. The trade winds are moist, as they have passed over
warm seas. Since they are easterlies, they cause greater precipitation on the eastern
coasts and run dry on the western coasts and interiors.
Q Source: Chapter 16: Goh Cheng Leong: Certificate Physical and Human
Geography

36 Consider the following about the use of language(s) in Parliament.


1. Assertion (A): A member of the house cannot address the House in his mother
tongue.
2. Reason (R): The Constitution has declared Hindi and English to be the languages for
transacting business in the Parliament.
In the context of the above, which of these is correct?
A. A is correct, and R is an appropriate explanation of A.
B. A is correct, but R is not an appropriate explanation of A

(C) Insights Active Learning. | All rights reserved.

www.insightsias.com

24

TEST - 22

User Name :

chandan paswan

Total Marks :

200

Mark Scored :

76

C. A is correct, but R is incorrect


D. A is incorrect, but R is correct
User Answer : D
Correct Answer : D
Answer Justification :

Learning: Hindi and English are to be the languages for transacting business in the
Parliament as per the constitution.

IA

However, the presiding officer can permit a member to address the House in his
mother tongue. In both the Houses, arrangements are made for simultaneous
translation.

TS

Learning: Though English was to be discontinued as a floor language after the


expiration of fifteen years from the commencement of the Constitution (that is, in
1965), the Official Languages Act (1963) allowed English to be continued along
with Hindi.

Q Source: Indian Polity: M Laxmikanth

IN
SI

37 Atacama Desert is the driest non-polar desert in the world where the mean annual
rainfall is not even an inch. This is due to
1. Presence of Cold Peruvian current on the Chilean coast
2. It being situated in the interior of South America
Which of the above is/are correct?
A. 1 only
B. 2 only
C. Both 1 and 2
D. None
User Answer : A
Correct Answer : A
Answer Justification :
Justification: Statement 1: The cold Peruvian current creates a region of high
pressure along the coasts. So, moisture laden winds from the Sea do not cross the
coasts and don't come to Atacama deserts.
Statement 2: The Atacama Desert is a plateau in South America, covering a 1,000kilometre strip of land on the Pacific coast, west of the Andes mountains. So, 2 is
clearly wrong.

(C) Insights Active Learning. | All rights reserved.

www.insightsias.com

25

TEST - 22

User Name :

chandan paswan

Total Marks

200

Mark Scored

76

Other reasons:

The Andes Range runs along the Atacama Desert, acting as a formidable
natural barrier from the moisture of the Amazon; the rain shadow effect.
The Atacama Desert is also located over high altitudes, above 2,500 metres
above sea level. This contributes to low drying temperatures and very low
humidity in the air.

IA

Q Source: Chapter 18: Goh Cheng Leong: Certificate Physical and Human
Geography

IN
SI

TS

38 The Atal Mission for Rejuvenation and Urban Transformation (AMRUT) focuses on
1. Ensuring that every household has access to a tap with assured supply of water and a
sewerage connection
2. Reducing pollution by switching to public transport and constructing facilities for
non-motorized transport
3. Developing places of religious character into international religious tourism hub
Which of the above is/are correct?
A. 1 and 2 only
B. 2 and 3 only
C. 1 only
D. 1 and 3 only
User Answer : C
Correct Answer : A
Answer Justification :
Justification: The Mission aims at providing basic services (e.g. water supply,
sewerage, urban transport) to households and build amenities in cities which will
improve the quality of life for all, especially the poor and the disadvantaged is a
national priority
It also aims at increasing the amenity value of cities by developing greenery and
well maintained pen spaces (e.g. parks).
The AMRUT makes States equal partners in planning and implementation of
projects, thus actualizing the spirit of cooperative federalism.
Statement 3 is the objective of HRIDAY scheme.
Q Source: Chapter 24: India Yearbook 2016

(C) Insights Active Learning. | All rights reserved.

www.insightsias.com

26

TEST - 22

User Name :

chandan paswan

Total Marks :

200

Mark Scored :

76

IA

39 It is the national tree of India. Its seeds are dispersed by fruit-eating birds. It is a big tree
and gives a lot of shades to traveller in very hot summer months. It is
A. Banyan Tree
B. Peepal Tree
C. Teak Tree
D. Neem Tree
User Answer : A
Correct Answer : A
Answer Justification :
Learning: In the Bhagavat Gita Krishna said "There is a banyan tree which has its
roots upward and its branches down, and the Vedic hymns are its leaves. One who
knows this tree is the knower of the Vedas."

TS

It is also called Indian or Bengal Fig. This tree is considered sacred in India and can
be seen near a temple or religious center.
The Banyan is part of the coat of arms of Indonesia. It is meant to symbolize the
unity of Indonesia - one country with many far-flung roots.

In Buddhism's Pali canon, the banyan, it is referenced numerous times.

Q Source: UPSC questions on national symbols (2015)

IN
SI

40 As per recently released World Health Organisation's (WHO) Urban Air Quality
Database for 2016, which is the most polluted Indian city?
A. Gwalior
B. Patna
C. Allahabad
D. Mumbai
User Answer : A
Correct Answer : A
Answer Justification :
Learning: According to a recent World Health Organization report the tourist
capital of Madhya Pradesh Gwalior tops the list. It is surrounded by three main
industrial and commercial areas: Sitholi, Banmore and Malanpur which accounts for
the rise in air pollution.
Half of the world's 20 most polluted cities are in India.

(C) Insights Active Learning. | All rights reserved.

www.insightsias.com

27

TEST - 22

User Name :

chandan paswan

Total Marks

200

Mark Scored

76

Other cities in the list are Allahabad, Patna, Delhi, Raipur etc.
Q Source:
http://www.hindustantimes.com/delhi/four-out-of-top-five-polluted-cities-are-in-indi
a-delhi-not-among-them/story-Gn2htcLbESB3BpeYJ4mY8K.html

TS

IA

41 The provision that "It is the duty of the Union to protect the states against external
aggression and internal disturbance" is a
A. Constitutional provision
B. Supreme Court Judgment
C. Parliamentary Enactment
D. Convention followed in a federal polity
User Answer : A
Correct Answer : A
Answer Justification :

Learning: As per A355, it is the duty of the Union to protect States against external
aggression and internal disturbance.

It shall be the duty of the Union to protect every State against external aggression
and internal disturbance and to ensure that the government of every State is carried
on in accordance with the provisions of this Constitution

IN
SI

While Article 356 has been repeatedly used and misused on flimsy grounds, the
Centre has seldom resorted to Article 355, because, though it serves as a precursor
to Article 356, it comes with an onerous burden
Q Source: Revision: Indian Polity: M Laxmikanth

42 Schedule I of Wildlife (Protection) Act, 1972 provides absolute protection to which of


the following species in India?
1. Lion Tailed Macaque
2. Vultures
3. Great Indian Bustard
4. Black Buck
Select the correct answer using the codes below
A. 1, 3 and 4 only
B. 2 and 4 only
C. 1 and 3 only
D. 1, 2, 3 and 4

(C) Insights Active Learning. | All rights reserved.

www.insightsias.com

28

TEST - 22

User Name :

chandan paswan

Total Marks :

200

Mark Scored :

76

User Answer : A
Correct Answer : A
Answer Justification :

You can see the list here

IA

http://www.moef.nic. in/legis/wildlife/wildlife2s1.pdf

Learning: Vultures are also protected, but the penalties are much less severe than in
case of Schedule I animals. They are covered under 4th schedule.

One needs to remember only important species.

TS

Q Source: Important Species: India

43 It is the largest national park of India and situated at a high altitude. The park is
bounded on the north by the banks of the Indus River, and includes the cachements of
Markha, Sumdah and Rumbak, and parts of the Zanskar Range. It is
A. Corbett National Park
B. Dachigam National Park
C. Hemis National Park
D. Great Himalayan National Park
User Answer : C
Correct Answer : C
Answer Justification :

IN
SI

Learning: It is located in the eastern Ladakh region. It is believed that highest


density of snow leopards of any protected area in the world occurs here.
The park lies within the Karakoram-West Tibetan Plateau alpine steppe ecoregion,
and contains pine forests, alpine shrublands and meadows, and alpine tundra
It is the only national park in India north of the Himalayas. So, this region is in the
rain shadow of the Himalayas, and thus does not receive much precipitation.
Hence, dry forests of juniper, Populus - Salix forests, subalpine dry birch - fir are
present at lower altitudes.
Q Source: https://en.wikipedia.org/wiki/Hemis_National_Park

44 The term 'Big Data' often in news is used in the context of data that is

(C) Insights Active Learning. | All rights reserved.

www.insightsias.com

29

TEST - 22

User Name :

chandan paswan

Total Marks

200

Mark Scored

76

A. Unsecured from cyber threats


B. Sensitive in nature and valued very high in markets
C. Posing a potential threat to national security
D. So large in volume that its analysis is exceedingly difficult
User Answer : D
Correct Answer : D
Answer Justification :

IA

Learning: Big data is being generated by everything around us at all times. Every
digital process and social media exchange produces it. Systems, sensors and mobile
devices transmit it

TS

Big data is arriving from multiple sources at an alarming velocity, volume and
variety. To extract meaningful value from big data, one needs optimal processing
power, analytics capabilities and skills
It can help us make better decisions. For e.g. weather forecasts can be made more
accurately if all the weather related data (precipitation, cloudiness, temperature,
humidity, dust storms, pressure systems) etc. be taken into analysis at one place

Q Source: http://www.dst.gov.in/big-data-initiative-1

IN
SI

45 Which of the following regions on earth qualify as biodiversity hotspots?


1. Areas where multiple ecological niches occur
2. Zones of High endemism
3. The biodiversity of the region is threatened
Select the correct answer using the codes below.
A. 1 and 3 only
B. 2 only
C. 2 and 3 only
D. 1 and 2 only
User Answer : A
Correct Answer : C
Answer Justification :
Learning: To qualify as a biodiversity hotspot, a region must meet two strict
criteria:
High endemism
It must have 30% or less of its original natural vegetation, i.e. threatened.

(C) Insights Active Learning. | All rights reserved.

www.insightsias.com

30

TEST - 22

User Name :

chandan paswan

Total Marks :

200

Mark Scored :

76

Around the world, 35 areas qualify as hotspots. They represent just 2.3% of Earth's
land surface, but they support more than half of the world's plant species as
endemics.

TS

IA

The diagram (taken from Wiki) shows the biodiversity hotspots of the world.

Q Source: ICSE Environmental Studies Xth

IN
SI

46 Which of the following states has the highest percentage of area under forest cover?
A. Mizoram
B. Meghalaya
C. Arunachal Pradesh
D. Madhya Pradesh
User Answer : D
Correct Answer : A
Answer Justification :
Learning: Among all the states and Union Territories, Mizoram has the highest
forest cover with 88.93 percent of the total area, followed by Lakshadweep.
States of Jammu and Kashmir, Uttarakhand, Meghalaya, Kerala, Arunachal Pradesh,
Karnataka and Telangana have suffered huge loss of forest cover
MP has the highest total area (not percentage) under forest cover, followed by
Arunachal Pradesh.

(C) Insights Active Learning. | All rights reserved.

www.insightsias.com

31

TEST - 22

User Name :

chandan paswan

Total Marks

200

Mark Scored

76

Q Source: India State Forest Report 2015

TS

IA

47 The rich growth of tropical vegetation can be attributed to which of the following
factors?
1. Hot and wet climate throughout the year
2. Fertile and nutrient rich soil
3. Frequent seasonal change helping in nutrient absorption from soil
Select the correct answer using the codes below
A. 1 and 2 only
B. 2 and 3 only
C. 1 only
D. 1, 2 and 3
User Answer : C
Correct Answer : C
Answer Justification :

Justification: Statement 1: Being close to the equator, solar insolation remains high
throughout the year causing more evaporation, rain and high humidity.

Statement 2: The soil is poor in nutrients and acidic due to frequent leaching by
heavy rains

Statement 3: Season remains more or less the same throughout the year since they
are very close to the equator

IN
SI

Q Source: Revision: 8th NCERT: Geography


48 The International Bill of Rights for Women comes from
A. The United Nations Charter
B. A resolution of the United Nations Human Rights Commission (UNHCR)
C. Convention on the Elimination of All Forms of Discrimination against
Women (CEDAW)
D. None of the above
User Answer :
Correct Answer : C
Answer Justification :
Learning: In 1979, the General Assembly adopted the Convention on the
Elimination of All Forms of Discrimination against Women (CEDAW), which is
often described as an International Bill of Rights for Women.

(C) Insights Active Learning. | All rights reserved.

www.insightsias.com

32

TEST - 22

User Name :

chandan paswan

Total Marks :

200

Mark Scored :

76

In its 30 articles, the Convention explicitly defines discrimination against women


and sets up an agenda for national action to end such discrimination
The Convention targets culture and tradition as influential forces shaping gender
roles and family relations, and it is the first human rights treaty to affirm the
reproductive rights of women

Q Source: Rights Issue: UPSC Syllabus

TS

IA

49 Which of the following would accelerate the process of climate change on earth?
1. Deforestation
2. Burning large amounts of fossil fuels
3. Reducing the capacity of oceans to absorb carbon dioxide
Select the correct answer using the codes below
A. 1 and 2 only
B. 2 and 3 only
C. 1 and 3 only
D. 1, 2 and 3
User Answer : D
Correct Answer : D
Answer Justification :

Justification: Climate change is caused on earth by:

IN
SI

Variations in the sun's energy reaching Earth


Changes in the reflectivity of Earth's atmosphere and surface
Changes in the greenhouse effect, which affects the amount of heat retained
by Earth's atmosphere

Carbon dioxide (CO2) is a minor but very important component of the atmosphere,
carbon dioxide is released through natural processes such as respiration and volcano
eruptions and through human activities such as deforestation, land use changes, and
burning fossil fuels.
Humans have increased atmospheric CO2 concentration by a third since the
Industrial Revolution began. This is the most important long-lived "forcing" of
climate change.
Oceans and forests absorb CO2. If their capacity to absorb carbon is reduced,
climate change will accelerate.

(C) Insights Active Learning. | All rights reserved.

www.insightsias.com

33

TEST - 22

User Name :

chandan paswan

Total Marks

200

Mark Scored

76

Q Source: ICSE Environmental Studies Xth

IA

50 Consider the following statements.


1. Assertion (A): The first Indian script of which types were prepared was Tamil.
2. Reason (R): Printing press was indigenously developed in India.
In the context of the above, which of these is correct?
A. A is correct, and R is an appropriate explanation of A.
B. A is correct, but R is not an appropriate explanation of A
C. A is correct, but R is incorrect
D. Both A and R are incorrect
User Answer : D
Correct Answer : C
Answer Justification :

TS

Justification & LearningThe art of printing first entered India through Goa.
In 1556, a Portuguese ship was put into Goa for victualling (actually setting sail to
Abyssinia).

The clergy in Goa felt their need for a printing press was greater than Abyssinia's
and, so requested the Governor-General to make the press available to them, and
thus printing was initiated in India serendipitously. So, R is incorrect.

IN
SI

The press was used by the Christian missionaries for printing Biblical propaganda
material
A Spaniard, Joao Gonsalves, is credited with preparing the first printing types of an
Indian script- Tamil
These types were used to print the first book in an Indian language in India (the first
Tamil book was printed in Lisbon in 1554 in Romanized Tamil script). So, A is
correct
You can read more here
http://www.thehindu.com/todays-paper/tp-features/tp-sundaymagazine/from-palm-le
aves-to-the-printed-word/article2275089.ece
Q Source: UPSC Past year papers
51 Mercosur, also known as the "Common Market of the South" is

(C) Insights Active Learning. | All rights reserved.

www.insightsias.com

34

TEST - 22

User Name :

chandan paswan

Total Marks :

200

Mark Scored :

76

A. An economic Union of Southern African states


B. A free trade area among the Central Asian states
C. An economic and political agreement among some South American states
D. A private trade body that acts as a representative of South Asia in WTO
User Answer :
Correct Answer : C
Answer Justification :

IA

Learning: It is a sub-regional bloc. Its full members are Argentina, Brazil,


Paraguay, Uruguay and Venezuela.

Its associate countries are Bolivia, Chile, Peru, Colombia, Ecuador and Suriname.
Observer countries are New Zealand and Mexico.

TS

Its purpose is to promote free trade and the fluid movement of goods, people, and
currency, as it is a full customs Union.
Q Source: Revision: International bodies

IN
SI

52 Which of the following is NOT a feature of the Companies Act, 2013?


A. It provides for mandatory corporate social responsibility (CSR) for certain
companies
B. There is a provision for at least one Woman Director in certain companies.
C. All decisions of a company will be taken unanimously by all its shareholders
D. All the above are features of the Companies Act.
User Answer :
Correct Answer : C
Answer Justification :
Justification: Decision is made by the Board of Directors, not by all the
shareholders. Certainly shareholders play an important role in decision making. So,
(c) is wrong.
Every Listed Company /Public Company with paid up capital of Rs 100 Crores or
more / Public Company with turnover of Rs 300 Crores or more shall have at least
one Woman Director.
Every company having net worth of rupees five hundred crore or more, or turnover
of rupees one thousand crore or more or a net profit of rupees five crore or more
during any financial year shall constitute a Corporate Social Responsibility
Committee of the Board consisting of three or more directors, out of which at least

(C) Insights Active Learning. | All rights reserved.

www.insightsias.com

35

TEST - 22

User Name :

chandan paswan

Total Marks

200

Mark Scored

76

one director shall be an independent director.


Q Source: UPSC past year papers + Often in news

TS

IA

53 Nitrous oxide is emitted when nitrogen is added to the soil through the use of chemical
fertilizers. But, how nitrous oxide is removed from the atmosphere?
1. Absorbed by certain types of bacteria
2. Destroyed by ultraviolet radiation
Which of the above is/are correct?
A. 1 only
B. 2 only
C. Both 1 and 2
D. None
User Answer : A
Correct Answer : C
Answer Justification :

Learning: Nitrous oxide emissions occur naturally through many sources associated
with the nitrogen cycle, which is the natural circulation of nitrogen among the
atmosphere, plants, animals, and microorganisms that live in soil and water.

IN
SI

Nitrogen takes on a variety of chemical forms throughout the nitrogen cycle,


including N2O. Natural emissions of N2O are mainly from bacteria breaking down
nitrogen in soils and the oceans. Nitrous oxide is removed from the atmosphere
when it is absorbed by certain types of bacteria or destroyed by ultraviolet radiation
or chemical reactions
Nitrous oxide has one of the longest atmosphere lifetimes of the greenhouse gases,
lasting for up to 150 years
Q Source: ICSE Environmental Studies Xth

54 Which of the following is INCORRECT about the Mughal school of painting?


A. It does not portray naturalistic themes.
B. It is aristocratic and secular in nature
C. It is closely influenced by Persian school of painting
D. None of the above
User Answer : A
Correct Answer : A
Answer Justification :

(C) Insights Active Learning. | All rights reserved.

www.insightsias.com

36

TEST - 22

User Name :

chandan paswan

Total Marks :

200

Mark Scored :

76

Justification & Learning: Emperor Akbar was keenly interested in the art of
painting and architecture.
A large number of Indian artists from all over India were recruited to work under the
Persian masters appointed by Akbar for the development of paintings.

IA

The Mughal style evolved as a result of a happy synthesis of the indigenous Indian
style of painting and the Safavid school of Persian painting. The Mughal style is
marked by supple naturalism based on close observation of nature and fine and
delicate drawing. It is of an high aesthetic merit. It is primarily aristocratic and
secular.
Q Source: http://www.ccrtindia.gov.in/visualarts.php

TS

55 It was bounded on the north by the river Ganges, on the east by the river Champa, on
the south by the Vindhya Range, and on the west by the Son River. It refers to
A. Nanda Empire
B. Magadha Empire
C. Susunga Empire
D. Anga Empire
User Answer : D
Correct Answer : B
Answer Justification :

IN
SI

Learning: The kingdom of the Magadha roughly corresponds to the modern


districts of Patna, Jehanabad, Nalanda, Aurangabad, Nawadah and Gaya in southern
Bihar, and parts of Bengal in the east.
The most important sources of information about Magadha are the Buddhist Pali
Canon, the Jain Agamas and the Hindu Puranas
Magadha expanded to include most of Bihar and Bengal with the conquest of
Licchavi and Anga, respectively, followed by much of eastern Uttar Pradesh and
Odisha. The ancient kingdom of Magadha is heavily mentioned in Jain and Buddhist
texts.
Q Source: Improvisation: River Kingdoms: UPSC 2015

56 A complex administrative apparatus was recommended by Kautilya to tackle corruption


in the Mauryan Empire. What was/were the mean(s) of checking corruption in the

(C) Insights Active Learning. | All rights reserved.

www.insightsias.com

37

TEST - 22

User Name :

chandan paswan

Total Marks

200

Mark Scored

76

IA

administration?
1. There was a bureau of espionage that acted as a deterrent for the corrupt.
2. Officials were transferred frequently
3. Different registers were kept to detect any embezzlement of treasury funds.
Select the correct answer using the codes below
A. 1 and 2 only
B. 3 only
C. 2 only
D. 1, 2 and 3
User Answer : C
Correct Answer : D
Answer Justification :

TS

Justification: Kautilya, who played a significant role in the design of Mauryan


administrative apparatus, knows that men are fickle, corruptible and unpredictable.
Therefore, he suggests preventive measures to keep employees on the path of
honesty and loyalty to the king.

IN
SI

To ensure the loyalty of government servants, he assigns a significant role to


the bureau of espionage, for the bureau can instil in them the fear of detection
and consequent punishment
Different registers were kept to facilitate checking and strict instructions laid
down for the detection of embezzlement. Frequent transfer of officials was
recommended as a means of checking corruption.
Kautilya prescribes techniques to ensure the integrity and loyalty of
administrative services.
The king is advised to be extremely generous to those officials who prove to
be above temptation and ruthlessly severe to those who misuse powers,
misappropriate funds or betray responsibilities.

Q Source: Ancient Indian History and Civilization by Sailendra Nath Sen

57 Which of the following is responsible for the mild weather experienced around Alaska's
southern coast and in British Columbia despite being situated at higher latitude?
A. Japan Current which is a warm ocean current
B. Drilling of oil in nearby regions
C. Marine upwelling that reduces surface temperatures
D. Release of carbondioxide by planktons that warms the nearby atmosphere
User Answer : C
Correct Answer : A

(C) Insights Active Learning. | All rights reserved.

www.insightsias.com

38

TEST - 22

User Name :

chandan paswan

Total Marks :

200

Mark Scored :

76

Answer Justification :
Justification: The Japan Current begins off the east coast of Taiwan and flows
north-eastward past Japan, where it merges with the easterly drift of the North
Pacific Current.

It is analogous to the Gulf Stream in the Atlantic Ocean, transporting warm, tropical
water northward towards the polar region.

IA

The warm waters of the Kuroshio Current sustain the coral reefs of Japan, the
northernmost coral reefs in the world, and they are also responsible for the higher
temperatures recorded near British Columbia.

TS

Q Source: Improvisation: UPSC past year Qs on Kuroshio current

58 Which of the following is a lethal heat resistant spoilage organism in the canning
industry?
A. E.coli
B. Prokaryota. C
C. Bacillus subtilis
D. Clostridium botulinum
User Answer :
Correct Answer : D
Answer Justification :

IN
SI

Learning: All of the sterilization treatments tend to affect adversely the material to
be preserved, and often only partial sterilization is used in order to minimize damage to the product
For e.g. pasteurization of milk and other products, such as beer, is aimed at the
destruction of heat-sensitive human pathogens and an overall reduction in microbial
numbers; the complete sterilization of milk and beer by heat would render them
unpalatable.
In the canning industry, the most dangerous and highly heat-resistant spoilage
organism is Clostridium botulinum.
Minute amounts of the bowfin neurotoxin produced by this spore-forming
anaerobe can cause rapid fatality by respiratory paralysis.
Canned food products that may serve as substrates for Clostridium and other
endospore-forming bacteria need extensive heat processing to eliminate all
viable endospores.

(C) Insights Active Learning. | All rights reserved.

www.insightsias.com

39

TEST - 22

User Name :

chandan paswan

Total Marks

200

Mark Scored

76

Foods that are by nature acidic (fruit preserves, toma-toes, and so on) are
satisfactorily preserved with a milder heat process. as C. botulinum fails to
grow and produce toxins at low pH.
Q Source: UPSC past year papers SCRA examination

TS

IA

59 Consider the following about the China-Pakistan Economic Corridor (CPEC) Project
1. It consists of transport as well as power projects.
2. It is intended to link the Xinjiang province of China to Gwadar deep sea port close
to Pakistan's border with Iran.
3. It is a part of China's 'One Belt One Road' (OBOR) initiative.
Select the correct answer using the codes below
A. 1 and 2 only
B. 2 and 3 only
C. 1 and 3 only
D. 1, 2 and 3
User Answer : D
Correct Answer : D
Answer Justification :

Learning: It refers to a clutch of major infrastructure projects currently under way


in Pakistan. The corridor is mainly consists of road, rail and power projects.

IN
SI

The CPEC is part of China's regional transnational 'One Belt One Road'
(OBOR) initiative that connects large patch of Asia and Eastern Europe.
Gwadar Port is strategically located and lies close to the Strait of Hormuz
(near Iran), a key international oil shipping lane.
It could open up an energy and trade corridor from the Gulf region across
Pakistan to western China and also could be used by the Chinese Navy for
military purpose
The CPEC will give China land access to the Indian Ocean, cutting the nearly
13,000 km sea voyage through the Strait of Malacca (major chokepoint) and
around India to 2,000 km road journey from Gwadar to Kashgar.
The development of Kashgar as a trade terminus will reduce isolation of the
underdeveloped Xinjiang province and deepen its engagement with the rest of
China

Q Source: Often in news


60 Consider the following statements

(C) Insights Active Learning. | All rights reserved.

www.insightsias.com

40

TEST - 22

User Name :

chandan paswan

Total Marks :

200

Mark Scored :

76

IA

1. It is produced from molasses, one of the by-products of Sugar industry.


2. The Government had decided its mandatory blending with petrol should be
implemented across the country
The above refers to?
A. Butanol
B. Bio-diesel
C. Bio-starch
D. Ethanol
User Answer : D
Correct Answer : D
Answer Justification :

TS

Learning: Ethanol is an agro-based product, basically produced from the byproduct of the sugar industry, viz. molasses. In the years of surplus production of
sugarcane, when the sugar prices are depressed, the sugar industry is unable to pay
cane price to the farmers

This is mainly due to surplus production of sugar. The ethanol blended petrol
programme, besides lowering pollution levels, is expected to provide another outlet
for ethanol use, thus insuring utilization of molasses produced as a by-product
during manufacture of sugar.

It was decided by the Government that 5 per cent mandatory ethanol blending with
petrol should be implemented across the country and procurement price of ethanol
will be decided between Oil Marketing Companies and suppliers of ethanol.

IN
SI

This is expected to generate revenue for sugar mills enabling them to avoid building
up of cane price arrears.
Q Source: 10th NCERT Geography

61 Which of the following statements is INCORRECT with reference to Nyaya


Panchayats?
1. They have been given statutory status under the Nyaya Panchayats Act 2009
2. They function directly under the State High court and report to it.
Select the correct answer using the codes below
A. 1 only
B. 2 only
C. Both 1 and 2
D. None
User Answer : D

(C) Insights Active Learning. | All rights reserved.

www.insightsias.com

41

TEST - 22

User Name :

chandan paswan

Total Marks

200

Mark Scored

76

Correct Answer : C
Answer Justification :
Justification: Statement 2: Legislation to formalise these bodies and bring them
within the ambit of organised justice in India was planned as part of the Panchayati
Raj reforms of Rajiv Gandhi in the 1980s.

IA

To draft legislation in this regard a drafting committee, under the chairmanship of


Professor Upendra Baxi, has been formed by the Ministry of Panchayati Raj,
Government of India. The bill on the issue is yet to be passed by the Parliament. So,
1 is incorrect.

TS

Learning: The earliest nyaya panchayats were the village courts established under
the Village Courts Act of 1888. The Royal Commission on Decentralisation of
1909, recommended of revival of nyaya panchayats having both civil and criminal
jurisdiction in petty cases arising within the village.
Q Source:Revision: Chapter on Law and Justice: India Yearbook 2016

IN
SI

62 Current transactions of an economy do NOT include


A. Export
B. Interest payments
C. Portfolio investment
D. Import
User Answer : B
Correct Answer : C
Answer Justification :

Learning: Current transactions of an economy in foreign currency all over the


world are-export, import, interest payments, foreign investment in shares
FII, FDI, portfolio investment form part of capital account.
All transactions are shown as either inflow or outflow (credit or debit). At the end of
the year, the current account might be positive or negative.
India had surplus current accounts for three consecutive years (2000-03)-the only
such period.
Q Source: 12th NCERT: Macroeconomics

(C) Insights Active Learning. | All rights reserved.

www.insightsias.com

42

TEST - 22

User Name :

chandan paswan

Total Marks :

200

Mark Scored :

76

IA

63 When domestic currency appreciates with respect to a foreign currency it benefits


domestic
1. Importers
2. Exporters
3. Corporate external borrowers
Select the correct answer using the codes below
A. 1 and 3 only
B. 1 only
C. 3 only
D. 2 and 3 only
User Answer : B
Correct Answer : B
Answer Justification :

TS

Justification: Statement 1: Importers benefit as foreign goods become cheaper to


buy.

Statement 2: Exporters suffer as now domestic export become costlier in


international markets and they lose their competitiveness. We have covered this
concept earlier too

Statement 3: When the local currency is appreciating the debt service liability is
falling in domestic currency terms for the corporate borrower

IN
SI

A sharp depreciation in local currency would mean corresponding increase in debt


service liability, as more domestic currency would be required to buy the same
amount of foreign exchange for debt.
Q Source: 12th NCERT: Macroeconomics

64 Base Rate is the interest rate below which


A. Scheduled Commercial Banks (SCBs) would not lend to consumers
B. SCBs would not borrow from RBI
C. RBI would not lend to SCBs
D. SCBs cannot give interest for the public bank deposits
User Answer : A
Correct Answer : A
Answer Justification :
Learning: It is like the Prime Lending Rate (PLR) and the Benchmark Prime
lending Rate (BPLR) of the past and is basically a floor rate of interest.

(C) Insights Active Learning. | All rights reserved.

www.insightsias.com

43

TEST - 22

User Name :

chandan paswan

Total Marks

200

Mark Scored

76

It replaced BPLR in 2010.


Base Rate system is aimed at enhancing transparency in lending rates of banks and
enabling better assessment of transmission of monetary policy.
This was because in the earlier system banks could lend below BPLR

IA

This allowed bargaining by the borrower with bank. So, ultimately one borrower
was getting cheaper loan than the other, and blurred the attempts of bringing in
transparency in the lending business.
For the same reason, it was also difficult to assess the transmission of policy rates
(i.e. repo rate, reverse repo rate, bank rate) of the Reserve Bank to lending rates of
banks.

TS

Q Source: Revision: Indian Economy: Ramesh Singh

IN
SI

65 The Fundamental Right to Property is still guaranteed in which of the following states
of India?
A. Jammu and Kashmir
B. Assam
C. Nagaland
D. Telangana
User Answer : A
Correct Answer : A
Answer Justification :
Learning: Part III (dealing with Fundamental Rights) is applicable to the state of
J&K with some exceptions and conditions. The Fundamental Right to Property is
still guaranteed in the state.
Also, certain special rights are granted to the permanent residents of the state with
regard to public employment, acquisition of immovable property, settlement and
government scholarships
Part IV (dealing with Directive Principles of State Policy) and Part IVA (dealing
with Fundamental Duties) are not applicable to the state.
Q Source: Indian Polity: M Laxmikanth
66 Major rock edicts (Ashokan inscriptions) record which of the following?

(C) Insights Active Learning. | All rights reserved.

www.insightsias.com

44

TEST - 22

User Name :

chandan paswan

Total Marks :

200

Mark Scored :

76

IA

1. Fame and glory that comes to a Kingdom as a result of war


2. Measures of social welfare
3. Prohibition of animal sacrifice
Select the correct answer using the codes below.
A. 1 only
B. 2 and 3 only
C. 1 and 3 only
D. 1, 2 and 3
User Answer : B
Correct Answer : B
Answer Justification :

TS

Justification:Dhamma is a set of edicts that formed a policy of the Mauryan


emperor Ashoka Maurya.
Statement 1: Major Rock Edict X denounces fame and glory and reasserts the merits
of following the policy of Dhamma

Statement 2: Major Rock Edict II relates to measures of social welfare. It mentions


medical treatment for men and animals, construction of roads, wells and tree
planting.

Statement 3: Major Rock Edict I prohibits animal sacrifice and holidays of festive
gathering.

IN
SI

Q Source:11th TamilNadu Textbook


67 Which of the following is NOT a compulsory provision under the 73rd constitutional
amendment Act?
A. 21 years to be the minimum age for contesting elections to panchayats.
B. Organisation of Gram Sabha in a village or group of villages.
C. Establishment of a State Election Commission for conducting elections to the
panchayats.
D. Establishment of District Planning Committee for consolidation of plans
prepared by the local bodies
User Answer : B
Correct Answer : D
Answer Justification :
Justification: Option (d) is a compulsory provision under the 74th constitutional
amendment Act.

(C) Insights Active Learning. | All rights reserved.

www.insightsias.com

45

TEST - 22

User Name :

chandan paswan

Total Marks

200

Mark Scored

76

Other important compulsory provisions are:

Q Source: Indian Polity: M Laxmikanth

IA

Constitution of a State Finance Commission after every five years to review


the financial position of the panchayats.
Reservation of one-third seats (both members and chairpersons) for women in
panchayats at all the three levels.
Fixing tenure of five years for panchayats at all levels and holding fresh
elections within six months in the event of supersession of any panchayat.

TS

68 The institution of high court originated in India when


A. Supreme Court took charge in Independent India and established High courts
in many states
B. The Morley-Minto Act provided for the establishment of High courts
C. During the British rule when the high courts were set up at Calcutta, Bombay
and Madras
D. The 1773 Regulating Act created the Supreme Court as well as the High
Courts
User Answer : C
Correct Answer : C
Answer Justification :

IN
SI

Learning:The institution of high court originated in India in 1862 when the high
courts were set up at Calcutta, Bombay and Madras.

In 1866, a fourth high court was established at Allahabad. In the course of


time, each province in British India came to have its own high court.
After 1950, a high court existing in a province became the high court for the
corresponding state.
The Constitution of India provides for a high court for each state, but the
Seventh Amendment Act of 1956 authorised the Parliament to establish a
common high court for two or more states or for two or more states and a
union territory.

Q Source: Indian Polity: M Laxmikanth


69 Under the National Action Plan for Climate Change (NAPCC), missions have been

(C) Insights Active Learning. | All rights reserved.

www.insightsias.com

46

TEST - 22

User Name :

chandan paswan

Total Marks :

200

Mark Scored :

76

IA

launched for specifically addressing which of the following areas?


1. Himalayan ecosystem
2. Enhanced Energy Efficiency
3. Strategic Knowledge for Climate Change
Select the correct answer using the codes below.
A. 1 and 2 only
B. 2 and 3 only
C. 1 and 3 only
D. 1, 2 and 3 only
User Answer :
Correct Answer : D
Answer Justification :

TS

Justification: Statement 1: The plan aims to conserve biodiversity, forest cover, and
other ecological values in the Himalayan region, where glaciers that are a major
source of India's water supply are projected to recede as a result of global warming.

Statement 2: Current initiatives are expected to yield savings of 10,000 MW by


2012. Building on the Energy Conservation Act 2001, the plan recommends:

Mandating specific energy consumption decreases in large energy-consuming


industries, with a system for companies to trade energy-savings certificates;
Energy incentives, including reduced taxes on energy-efficient appliances etc.

IN
SI

Statement 3: To gain a better understanding of climate science, impacts and


challenges, the plan envisions a new Climate Science Research Fund, improved
climate modeling, and increased international collaboration. It also encourage
private sector initiatives to develop adaptation and mitigation technologies through
venture capital funds.
Four new missions are proposed to be added. Details are here
http://indianexpress.com/article/india/india-others/four-new-missions-to-boost-respo
nse-to-climate-change/
Q Source:Chapter 25: India Yearbook 2016

70 Wall paintings can be found in


1. Bhimbetka
2. Ajanta

(C) Insights Active Learning. | All rights reserved.

www.insightsias.com

47

TEST - 22

User Name :

chandan paswan

Total Marks

200

Mark Scored

76

IA

3. Bagh caves
Select the correct answer using the codes below.
A. 1 only
B. 1 and 3 only
C. 2 and 3 only
D. 1, 2 and 3 only
User Answer : D
Correct Answer : D
Answer Justification :

TS

Justification: Statement 1: The rock shelters and caves of Bhimbetka have a large
number of paintings. The oldest paintings are considered to be 30,000 years old, but
some of the geometric figures date to as recently as the medieval period. The colours
used are vegetable colors which have endured through time
Statement 2: Buddhist themes have been wall painted in Ajanta, for e.g. Padmapani,
Bodhisattva etc.

Statement 3: The paintings from Bagh caves in Madhya Pradesh correspond to some
paintings of Ajanta.

Q Source:http://www.ccrtindia.gov.in/visualarts.php

IN
SI

71 Consider the following statements.


1. Assertion (A): The minimum and maximum strength of the Legislative assembly of
each state is fixed by the State law.
2. Reason (R): The legislative assembly mainly consists of representatives directly
elected by the people on the basis of universal adult franchise.
In the context of the above, which of these is correct?
A. A is correct, and R is an appropriate explanation of A.
B. A is correct, but R is not an appropriate explanation of A
C. A is correct, but R is incorrect.
D. A is incorrect, but R is correct.
User Answer : D
Correct Answer : D
Answer Justification :
Learning: Its maximum strength is fixed at 500 and minimum strength at 60. It
means that its strength varies from 60 to 500 depending on the population size of the
state.

(C) Insights Active Learning. | All rights reserved.

www.insightsias.com

48

TEST - 22

User Name :

chandan paswan

Total Marks :

200

Mark Scored :

76

However, in case of Arunachal Pradesh, Sikkim and Goa, the minimum number is
fixed at 30 and in case of Mizoram and Nagaland, it is 40 and 46 respectively.
Further, some members of the legislative assemblies in Sikkim and Nagaland are
also elected indirectly.

Q Source: Indian Polity: M Laxmikanth

TS

IA

72 In the election of the President, every elected member of either House of Parliament
shall have such number of votes as may be obtained by dividing the
A. Total number of votes assigned to MLAs of the states by the total number of
the elected MPs
B. Total population of India by the total number of seats in all the State
legislative assemblies
C. Total votes assigned to the President by the total number of seats in
Parliament
D. Total strength of Parliament by the number of nominations filed for the
President's office
User Answer : A
Correct Answer : A
Answer Justification :

IN
SI

Learning:Every elected member of the legislative assembly of a state shall have as


many votes as there are multiples of one thousand in the quotient obtained by
dividing the population of the state by the total number of the elected members of
the assembly.
Every elected member of either House of Parliament shall have such number of
votes as may be obtained by dividing the total number of votes assigned to members
of the legislative assemblies of the states by the total number of the elected members
of both the Houses of Parliament
The President's election is held in accordance with the system of proportional
representation and single transferrable vote.
Q Source: Indian Polity: M Laxmikanth
73 Consider the following statements.
1. It is a rock-cut cave.
2. It is located in the Barabar and Nagarjuni hills.

(C) Insights Active Learning. | All rights reserved.

www.insightsias.com

49

chandan paswan

Total Marks

200

Mark Scored

76

IA

3. It was excavated for the Ajivika sect in the time of Ashoka.


The above refer to?
A. Lomus Rishi caves
B. Lenyadri Caves
C. Kanheri Caves
D. Badami Caves
User Answer : A
Correct Answer : A
Answer Justification :

TEST - 22

User Name :

Learning:Lomas Rishi Cave is carved into the hard monolithic granite rock face of
Barabar hills, flanked to its left by the smaller Sudama cave.

TS

The hut-style facade at the entrance to the cave, which was meant as a milk shed,
had an impact on South Asian rock-cut architecture. It became a model for all such
arched entrance portals built at many other Buddhist and Jain caves in India.
The ornamentation on the "curved architrave" consists of carvings of elephants on
their way to the stupas

Q Source:http://www.ccrtindia.gov.in/induscivilization.php

IN
SI

74 Mars Orbiter Mission is ISRO's first interplanetary mission to planet Mars. What will it
help us understand?
1. Possibility or the past existence of life on planet Mars
2. Role of Martian corona in affecting long-term climate change on earth
Which of the above is/are correct?
A. 1 only
B. 2 only
C. Both 1 and 2
D. None
User Answer : A
Correct Answer : A
Answer Justification :
Justification:MOM will explore and observe Mars surface features, morphology,
mineralogy and the Martian atmosphere. Further, a specific search for methane in
the Martian atmosphere will provide information about the possibility or the past
existence of life on the planet.
The enormous distances involved in interplanetary missions present a demanding

(C) Insights Active Learning. | All rights reserved.

www.insightsias.com

50

TEST - 22

User Name :

chandan paswan

Total Marks :

200

Mark Scored :

76

challenge; developing and mastering the technologies essential for these missions
will open endless possibilities for space exploration.
Q Source:Chapter 25: India Yearbook 2016

IA

75 The Savanna climate is experienced best in Sudan. Why?


A. Dry and wet seasons are most distinct in Sudan.
B. Trade winds do not alter the climate of Sudan.
C. There is no wet season other than occasional precipitation spurts in Sudan.
D. Due to its small geographical extent
User Answer : A
Correct Answer : A
Answer Justification :

TS

Learning:It is a transitional type of climate between the equatorial forest and the
trade wind hot deserts.

It is confined within the tropics and is best developed in the Sudan where the dry
and wet seasons are most distinct, hence its name the Sudan climate.

The belt includes West African Sudan, and then curves southward into east Africa
and southern Africa north of the tropic of Capricorn.
In South America, there are two distinct regions of savannah north and south of the
equator.

IN
SI

Q Source: Chapter 17: Goh Cheng Leong: Certificate Physical and Human
Geography

76 Which of the following statements about the Calling Attention Motion is


INCORRECT?
A. It can be introduced only in Lok Sabha.
B. The motion seeks an authoritative statement from the Minister concerned on
that matter.
C. Like the zero hour, it is also an Indian innovation in the parliamentary
procedure.
D. It is mentioned in the Rules of Procedure.
User Answer :
Correct Answer : A
Answer Justification :

(C) Insights Active Learning. | All rights reserved.

www.insightsias.com

51

TEST - 22

User Name :

chandan paswan

Total Marks

200

Mark Scored

76

Justification:It can be introduced in both the houses.


A member may, with the previous permission of the Speaker, call the attention of a
Minister to any matter of urgent public importance and the Minister may make a
brief statement or ask for time to make a statement at a later hour or date.

It is the Censure motion and no-Confidence motion that can be moved only in Lok
Sabha.

IA

Q Source:Indian Polity: M Laxmikanth

TS

77 Consider the following about Contingency Fund of India.


1. The fund is held by the finance secretary on behalf of the president.
2. Parliamentary approval is not required in utilizing the fund as it falls outside the
ambit of Parliament.
Which of the above is/are correct?
A. 1 only
B. 2 only
C. Both 1 and 2
D. None
User Answer : D
Correct Answer : A
Answer Justification :

IN
SI

Justification:The Constitution authorised the Parliament to establish a 'Contingency


Fund of India', into which amounts determined by law are paid from time to time.
Accordingly, the Parliament enacted the contingency fund of India Act in 1950. This
fund is placed at the disposal of the president, and he can make advances out of it to
meet unforeseen expenditure pending its authorisation by the Parliament.
The fund is held by the finance secretary on behalf of the president. Like the public
account of India, it is also operated by executive action.
Q Source:Indian Polity: M Laxmikanth
78 Which of the following is NOT a part of the Collective privileges of each house of
Parliament?
A. The courts are prohibited to inquire into the proceedings of a House or its
committees.

(C) Insights Active Learning. | All rights reserved.

www.insightsias.com

52

TEST - 22

User Name :

chandan paswan

Total Marks :

200

Mark Scored :

76

Learning:Other privileges are:

IA

B. The house can make rules to regulate its own procedure and the conduct of its
business and to adjudicate upon such matters.
C. The house can punish members as well as outsiders for breach of its
privileges or its contempt by even suspension and imprisonment.
D. It has the right to transfer to itself a case pending in the Judiciary that is
concerned with a member of that house.
User Answer : D
Correct Answer : D
Answer Justification :

TS

It has the right to publish its reports, debates and proceedings and also the
right to prohibit others from publishing the same.
It can punish members as well as outsiders for breach of its privileges or its
contempt by reprimand, admonition or imprisonment (also suspension or
expulsion, in case of members).
It has the right to receive immediate information of the arrest, detention,
conviction, imprisonment and release of a member.
It can institute inquiries and order the attendance of witnesses and send for
relevant papers and records.
No person (either a member or outsider) can be arrested, and no legal process
(civil or criminal) can be served within the premises of the House without the
permission of the presiding officer.

IN
SI

Q Source: Indian Polity: M Laxmikanth

79 Which one of the following states has the longest coastline?


A. Gujarat
B. Maharashtra
C. Tamilnadu
D. Andhra Pradesh
User Answer : D
Correct Answer : A
Answer Justification :
Learning:Total length of Indian coastline is 7517 Km. Out of this total length the
coastal length of mainland India is 5423 Km and the coastal length of all Islands
(Andaman & Nicobar, Kavaratti islands) is 2094 Km.

(C) Insights Active Learning. | All rights reserved.

www.insightsias.com

53

TEST - 22

User Name :

chandan paswan

Total Marks

200

Mark Scored

76

Gujarat's coastline is 1600 Km, followed by Tamilnadu, AP and Maharashtra.


Goa has the shortest coastline among all states.
Q Source:UPSC past year papers

TS

IA

80 Lichens are generally the first community to colonize a bare rock. This is because
A. They can withstand dry conditions.
B. They do not require nutrients for growth.
C. Lichens consume on Mosses.
D. Lichens survive on pollen grains blown by the Wind.
User Answer :
Correct Answer : A
Answer Justification :
Learning: Succession takes place in a series of stages, each called a sere.

At each stage, certain species can be identified which change the environment so
that it becomes more suitable for other species.

IN
SI

Imagine an area of bare rock. One of the few kinds of organism capable of
surviving on such an inhospitable area is lichen.
Lichen is therefore a pioneer species. As a symbiotic relationship between an
alga and a fungus, a lichen can survive considerable drying out.
In time, weathering of this base rock produces sand or soil, although in itself
this cannot support other plants.
However, as the lichens die and decompose they release sufficient nutrients
to support a community of small plants.
Mosses are typically the next stage in succession, followed by ferns. With the
continuing erosion of the rock and the increasing amount of organic matter
available from these plants, a thicker layer of soil is built up.
This then supports small flowering plants such as grasses and, by turn, shrubs
and trees.

Q Source:ICSE Environmental Studies Xth


81 In the Red Data book published by IUCN, there are some 'Pink' and 'Green' pages which
cover species that are
A. Extinct and Formerly endangered respectively

(C) Insights Active Learning. | All rights reserved.

www.insightsias.com

54

TEST - 22

User Name :

chandan paswan

Total Marks :

200

Mark Scored :

76

B. Formerly endangered and Least concern species respectively


C. Critically endangered and Formerly endangered respectively
D. National Animals and Least concern species respectively
User Answer :
Correct Answer : C
Answer Justification :

IA

Learning:The Red data book is the record of the list of Animals and plants which
are facing the risk of danger. It contains, Red, Pink and Green pages.
Red is symbolic of the danger that some species of both plants and animals presently
experience throughout the globe.
The Pink pages in the Red data book include the critically endangered species.

TS

Green pages are used for those species that were formerly endangered, but have now
recovered to a point where they are no longer threatened.

Q Source: ICSE Environmental Studies Xth

IN
SI

82 The securities market regulator Securities and Exchange Board of India (SEBI) has
tightened the Participatory Notes (P-note) norms. The main purpose of this decision is to
keep
A. Vigil on foreign investments to curb black money inflows in the country.
B. A track on the global remittance flows to India.
C. Vigil on production of counterfeit notes in India.
D. Foreign printed notes from coming into circulation in India.
User Answer : A
Correct Answer : A
Answer Justification :
Learning:Participatory Notes are offshore/overseas derivative instruments (ODIs)
issued by registered foreign institutional investors (FII) to overseas investors.
P-Notes are issued to overseas investors who wish to invest in the Indian stock
markets without registering themselves (anonymous) with the market regulator
SEBI.
P-Notes are used by money launderers. They first take funds out of country through
hawala and then get it back using P-Notes.

(C) Insights Active Learning. | All rights reserved.

www.insightsias.com

55

TEST - 22

User Name :

chandan paswan

Total Marks

200

Mark Scored

76

So, the main purpose of this decision is to keep vigil on foreign investments to curb
black money inflows in the country.
In this regard, SEBI has introduced Know Your Client (KYC) compliance for
holders of these instruments in order to bring them on a par with domestic investors.
SEBI also has sought information on the ultimate beneficiaries of these products.

IA

Q
Source:http://www.thehindu.com/business/new-sebi-norms-on-pnotes-make-investo
rs-squirm/article8625357.ece

TS

83 It is a large majestic migratory bird that breeds and winters in wetlands. In India, they
were known to winter at Keoladeo National Park. It refers to
A. Bengal Florican
B. White Bellied Heron
C. Siberian Crane
D. Great Indian Bustard
User Answer : C
Correct Answer : C
Answer Justification :

Learning:Their breeding population is in the Arctic tundra of western and eastern


Russia

IN
SI

The eastern populations migrate during winter to China while the western
population winters in Iran and formerly, in India and Nepal. Among the cranes, they
make the longest distance migrations.
The last recorded observation of Siberian Cranes from the central Asian flock was in
2002. They have been declared as critically endangered by IUCN.
Q Source: ICSE Environmental Studies Xth

84 Conservation reserves and community reserves are the outcomes of amendments made
to the Wild Life Protection Act in 2003. Who can declare community reserves?
A. State governments in consultation with the local people
B. Panchayats in consultation with the local community
C. Panchayats after taking permission from the State government
D. Central government after taking into account the suggestions from the local
community

(C) Insights Active Learning. | All rights reserved.

www.insightsias.com

56

TEST - 22

User Name :

chandan paswan

Total Marks :

200

Mark Scored :

76

User Answer :
Correct Answer : A
Answer Justification :

Learning: Conservation reserves and community reserves in India are terms


denoting protected areas of India which typically act as buffer zones to or
connectors and migration corridors between established national parks, wildlife
sanctuaries and reserved and protected forests of India.

IA

Such areas are designated as conservation areas if they are uninhabited and
completely owned by the Government of India but used for subsistence by
communities, and community areas if part of the lands are privately owned.

TS

Administration of such reserves would be through local people and local agencies
like the gram panchayat. But, they are declared by the State government.
Q Source:ICSE Environmental Studies Xth

IN
SI

85 Wetlands are said to be one of the largest sources of the Greenhouse gas, Methane.
What in the wetlands releases methane?
A. Bacteria that decompose organic material in the absence of oxygen
B. Roots of the plants that take up methane from the soil.
C. Nutrient imbalance releases methane.
D. Both (a) and (b)
User Answer : D
Correct Answer : D
Answer Justification :
Learning:Wetlands are characterized by water-logged soils and distinctive
communities of plant and animal species that have evolved and adapted to the
constant presence of water. Due to this high level of water saturation as well as
warm weather, wetlands are one of the most significant natural sources of
atmospheric methane.
Most methanogenesis, or methane production, occurs in oxygen poor environments.
Because the microbes that live in warm, moist environments consume oxygen more
rapidly than it can diffuse in from the atmosphere, wetlands are the ideal anaerobic,
or oxygen poor, environments for fermentation.
Plant aerenchyma refers to the vessel-like transport tubes within the tissues of
certain kinds of plants. Plants with arenchyma possess porous tissue that allows for

(C) Insights Active Learning. | All rights reserved.

www.insightsias.com

57

TEST - 22

User Name :

chandan paswan

Total Marks

200

Mark Scored

76

direct travel of gases to and from the plant roots. Methane can travel directly up
from the soil into the atmosphere using this transport system.
Q Source:ICSE Environmental Studies Xth

IA

86 Largest and smallest states of India in terms of geographical area respectively are
A. Rajasthan and Sikkim
B. Madhya Pradesh and Goa
C. Maharashtra and Sikkim
D. Rajasthan and Goa
User Answer : B
Correct Answer : D
Answer Justification :

TS

Learning:Smallest UT is Lakshadweep. Daman and Diu is the second smallest UT.


Sikkim is second smallest state, followed by Tripura and Nagaland.

MP is the second largest state. Maharashtra is the third largest, followed by UP and
Gujarat.

Q Source:General facts: Often asked in UPSC (as in 2015)

IN
SI

87 A biosphere Reserve is demarcated in 3 regions - Core, Buffer and Transition Zone.


Consider the following about it.
1. The core zone is generally kept free from human pressures external to the system.
2. Tourism, fishing and grazing can be permitted in the Buffer zone.
3. Settlements and crop lands can be found in transition zones.
Select the correct answer using the codes below.
A. 1 and 2 only
B. 2 and 3 only
C. 1 and 3 only
D. 1, 2 and 3 only
User Answer :
Correct Answer : D
Answer Justification :
Justification:Statement 1: Core zone must contain suitable habitat for numerous
plant and animal species, including higher order predators and may contain centres
of endemism. Core areas often conserve the wild relatives of economic species and

(C) Insights Active Learning. | All rights reserved.

www.insightsias.com

58

TEST - 22

User Name :

chandan paswan

Total Marks :

200

Mark Scored :

76

also represent important genetic reservoirs having exceptional scientific interest.


Statement 2: The buffer zone, adjoins or surrounds core zone, uses and activities are
managed in this area in the ways that help in protection of core zone in its natural
condition.

IA

These uses and activities include restoration, demonstration sites for enhancing
value addition to the resources, limited recreation, tourism, fishing, grazing, etc;
which are permitted to reduce its effect on core zone.
Research and educational activities are to be encouraged. Human activities, if
natural within BR, are likely to continue if these do not adversely affect the
ecological diversity.

TS

Statement 3: The transition area is the outermost part of a biosphere reserve. This is
usually not delimited one and is a zone of cooperation where conservation
knowledge and management skills are applied and uses are managed in harmony
with the purpose of the biosphere reserve.

This includes settlements, crop lands, managed forests and area for intensive
recreation and other economic uses characteristics of the region.

Q Source:ICSE Environmental Studies Xth

IN
SI

88 The Technology Development Board (TDB) as a part of the Department of Science and
Technology aims at
1. Supporting research universities in India meet quality standards
2. Accelerating the development and commercialisation of indigenous technology
3. Safeguard sensitive defence technology from being patented abroad by vested
interests
Select the correct answer using the codes below.
A. 1 only
B. 2 and 3 only
C. 2 only
D. 1 and 3 only
User Answer :
Correct Answer : C
Answer Justification :
Learning: The Technology Development Board is the first organization of its kind
within the Government framework with the sole objective of translating the fruits of

(C) Insights Active Learning. | All rights reserved.

www.insightsias.com

59

TEST - 22

User Name :

chandan paswan

Total Marks

200

Mark Scored

76

indigenous research into commercial products or services.


The Board plays a pro-active role by encouraging commercial enterprises to take up
technology oriented projects
The board provides financial assistance in the form of Equity, Soft loans, or Grants.

TDB provides equity capital or loans to industrial concerns and financial assistance
to research and development institutions.

IA

Q Source: http://www.dst.gov.in/technology-development-board

IN
SI

TS

89 The Biodiversity hotspots present in India are located in


1. North-western Himalayas
2. Eastern Ghats
3. Western Ghats
4. Indo-Burma region
Select the correct answer using the codes below.
A. 1 and 3 only
B. 3 and 4 only
C. 1, 2 and 4 only
D. 2, 3 and 4 only
User Answer : B
Correct Answer : B
Answer Justification :

Learning:It is the eastern Himalayas, Indo-Burma, Western Ghats and Sri Lanka
region that are the biodiversity hotspots in India.
North-western Himalayas (rain shadow region) is dry and does not host rich
biodiversity. So, 1 is wrong.
Eastern Ghats is discontinuous and does not receive high precipitation like Western
Ghats to host rich biodiversity. So, 2 is wrong.
Q Source:ICSE Environmental Studies Xth

90 Which of the following category of sites can be recognized as UNESCO World


Heritage sites?
A. Regions that are outstanding examples of earth's record of life or its

(C) Insights Active Learning. | All rights reserved.

www.insightsias.com

60

TEST - 22

User Name :

chandan paswan

Total Marks :

200

Mark Scored :

76

geological processes
B. Habitats hosting rare, endangered and exceptional biodiversity
C. Any human made structure that has survived over a thousand years and is still
conserved by a national government
D. Both (a) and (b)
User Answer : D
Correct Answer : D
Answer Justification :

IA

Learning: A World Heritage Site is a place (such as a building, city, complex,


desert, forest, island, lake, monument, or mountain) that is listed by the United
Nations Educational, Scientific and Cultural Organization (UNESCO) as being of
special cultural or physical significance.

TS

The criteria can be found here


http://whc.unesco.org/en/criteria/

While each World Heritage Site remains part of the legal territory of the state
wherein the site is located, UNESCO considers it in the interest of the international
community to preserve each site.

Q Source:ICSE Environmental Studies Xth

IN
SI

91 Trade and cultural contacts existed between Indus civilization cities and those of
Mesopotamia; it is evidenced by
A. Presence of similar housing pattern in both cities
B. Occurrence of similar seals and beads in both places
C. Discovery of Indus valley texts that discuss trade figures
D. All of the above
User Answer : B
Correct Answer : B
Answer Justification :
Justification:Along with ancient Egypt and Mesopotamia, Indus valley was one of
three early civilisations of the Old World, and of the three most widespread.
Judging from the dispersal of Indus civilisation artefacts, the trade networks,
economically, integrated a huge area, including portions of Afghanistan, the coastal
regions of Persia, northern and western India, and Mesopotamia

(C) Insights Active Learning. | All rights reserved.

www.insightsias.com

61

TEST - 22

User Name :

chandan paswan

Total Marks

200

Mark Scored

76

For e.g. the occurrence of the seals, as well as similar carnelian beads, knobbed
pottery, etc. at both places is one concrete evidence.
There was an extensive maritime trade network operating between the Harappan and
Mesopotamian civilisations.

Q Source:http://www.ccrtindia.gov.in/induscivilization.php

TS

IA

92 The idea of conserving endangered species by translocation to other protected areas has
often been in news. What is/are the disadvantages of concentrating all the population of a
species into a single protected area?
1. It exposes the species to the risks of calamities like floods and epidemics.
2. The protected area may overshoot its carrying capacity.
Which of the above is/are correct?
A. 1 only
B. 2 only
C. Both 1 and 2
D. None
User Answer :
Correct Answer : C
Answer Justification :

IN
SI

Justification:Statement 1: For e.g. concentrating a lot of Rhinos in a protected area


like Kaziranga can be disastrous if a flood were to come. If the Rhino population
were dispersed, such risks can be diversified.
Statement 2: The carrying capacity of a biological species in an environment is the
maximum population size of the species that the environment can sustain
indefinitely, given the food, habitat, water, and other necessities available in the
environment. Naturally, if the population exceeds the carrying capacity, the
ecosystem is bound to come in certain problems.
Q
Source:http://www.livemint.com/Politics/niSTqjTWnYXmlhogS55tuM/Gir-LionsForest-ministry-cites-challenges-in-Madhya-Prades.html

93 Which of the following statements with respect to 'Make in India' is correct?


1. It is being implemented by Ministry of Finance.
2. A body 'Invest India' has been created to help foreign investors sail through red tape.
3. It identifies domestic companies with leadership in innovation to make them global

(C) Insights Active Learning. | All rights reserved.

www.insightsias.com

62

TEST - 22

User Name :

chandan paswan

Total Marks :

200

Mark Scored :

76

IA

champions of technology and innovation through state support.


Select the correct answer using the codes below.
A. 1 and 2 only
B. 3 only
C. 2 and 3 only
D. 1, 2 and 3
User Answer : B
Correct Answer : A
Answer Justification :

Justification & Learning: It was launched by the Government of India to


encourage multi-national, as well as national companies to manufacture their
products in India.

TS

For companies setting up factories, "Invest India" unit is being set-up in the
commerce department which would be available 24/7.

The main focus of this department would be to make doing business in India easy by
making all the approval processes simpler and resolving the issues in getting
regulatory clearances within 48-72 hours so that clearances are fast.
You can read more at the Q Source.

Q Source:http://www.makeinindia.com/about

IN
SI

94 Consider the following statements.


1. It is the oldest port of India.
2. It is the only riverine major port of India.
3. It has a vast hinterland comprising the entire eastern India and the landlocked
countries of Nepal and Bhutan.
The above refer to which port of India?
A. Paradip
B. Haldia
C. Kolkata
D. Mormugao
User Answer : C
Correct Answer : C
Answer Justification :
Learning:Among major ports, Mumbai is the biggest. Kandla is a tidal port.

(C) Insights Active Learning. | All rights reserved.

www.insightsias.com

63

TEST - 22

User Name :

chandan paswan

Total Marks

200

Mark Scored

76

Marmugao enjoys the second position by value of the tonnage of the bulk of which
is export of Iron core.
Vishakhapatnam is the deepest land-locked and protected port.

Chennai has an artificial harbour, Kolkata is a riverine port, Haldia has a fully
equipped containerised berth.

IA

Q Source:Chapter 26: India Yearbook 2016

TS

95 Why 'Sacred Groves' hold an important place in the design of environmental


conservation policy in India?
A. It shows the importance of community-led conservation.
B. It shows how remote forests are better preserved than open forests.
C. It shows how legally protected forests do better than non-protected forests.
D. It shows how grant of land rights to tribals can result in better conservation.
User Answer :
Correct Answer : A
Answer Justification :

Justification & Learning:Sacred Groves are relic forest patches traditionally


protected by communities in reverence of a deity.

IN
SI

n absence of statutory protected areas and in the wake of mass deforestation in some
parts of India, Sacred Groves form important repositories of forest biodiversity and
provide refuge to many plant and animal species of conservation significance. India
has well over 13,000 documented Sacred Groves.

Almost every village in the Sahaydri-Konkan region (north Western Ghats)


has at least one Sacred Grove ranging from just a few acres to hundreds of
acres.
Sacred Groves are not only important sites for regional biodiversity but also
provide vital ecosystem services to local people.
Often streams and rivers originate from Sacred Groves or wells and tanks are
seen within or near the groves which form important water source for the
rural population.
These community forests harbor valuable biodiversity and are under
tremendous pressure and threats such as encroachments for agriculture,
grazing, developmental activities like roads, dams, canals, urbanization and
industrialization.

(C) Insights Active Learning. | All rights reserved.

www.insightsias.com

64

TEST - 22

User Name :

chandan paswan

Total Marks :

200

Mark Scored :

76

Q Source:ICSE Environmental Studies Xth

TS

IA

96 Article 143 of the Constitution authorises the president to seek the opinion of the
Supreme Court on any question of law or fact of public importance which is likely to arise.
Consider the following about it.
1. The Supreme Court is bound to given its advice to the President.
2. The advice of the SC is binding on the President and thus the Government.
Which of the above is/are correct?
A. 1 only
B. 2 only
C. Both 1 and 2
D. None
User Answer : C
Correct Answer : D
Answer Justification :
Justification: Two categories are covered by A143:

On any question of law or fact of public importance which has arisen or


which is likely to arise.
On any dispute arising out of any pre-constitution treaty, agreement,
covenant, engagement or other similar instruments.

IN
SI

In the first case, the Supreme Court may tender or may refuse to tender its opinion to
the president.
But, in the second case, the Supreme Court 'must' tender its opinion to the president.
In both the cases, the opinion expressed by the Supreme Court is only advisory and
not a judicial pronouncement.
Q Source:Indian Polity: M Laxmikanth

97 Which of the following factors were involved in the immediate breakout of the Quit
India Movement in 1942?
A. The Cripps mission had failed.
B. People were suffering from acute food shortages and high food inflation due
to famine.
C. British withdrawal of only British citizens from the ongoing war in South-

(C) Insights Active Learning. | All rights reserved.

www.insightsias.com

65

TEST - 22

User Name :

chandan paswan

Total Marks

200

Mark Scored

76

east Asia angered the Indians.


D. All of the above
User Answer : A
Correct Answer : D
Answer Justification :

IA

Learning:The Quit India movement was not a meticulously planned movement as


the previous mass movements.

TS

The masses were already frustrated with the British not giving them the right
to self-govern. This became clear with the failure of the Cripps Mission
which only offered a dominion status for India.
The incoming famine made people even more impatient that raised food
prices and caused food shortage.
Further, the news came in that the British have left Indian subjects to die in
the war at Burma. But, they had evacuated British citizens.
This infuriated the leaders and the masses. In their opinion, if there was
invasion from the side of Burma, this would be the fate of Indians too. The
British would simply flee.
All these led to the Quit India movement.

Q Source:India's struggle for Independence: Bipin Chandra

IN
SI

98 Black Carbon is a solid particle or aerosol that contributes to the warming of


atmosphere. How?
1. It is a strong absorber of sunlight and heats the air indirectly.
2. It darkens snow-packs and glaciers through deposition leading to their melting
Which of the above is/are correct?
A. 1 only
B. 2 only
C. Both 1 and 2
D. None
User Answer : A
Correct Answer : C
Answer Justification :
Justification & Learning:Black carbon, commonly known as soot, is a form of
particulate air pollution, produced from incomplete combustion from biomass
burning, cooking with solid fuels, and diesel exhaust.

(C) Insights Active Learning. | All rights reserved.

www.insightsias.com

66

TEST - 22

User Name :

chandan paswan

Total Marks :

200

Mark Scored :

76

It consists of pure carbon in several linked forms.

IA

Black carbon warms the Earth by absorbing heat in the atmosphere and by
reducing albedo, the ability to reflect sunlight, when deposited on snow and
ice.
BC is the strongest absorber of sunlight and heats the air directly. In addition,
it darkens snow packs and glaciers through deposition and leads to melting of
ice and snow.
Regionally, BC disrupts cloudiness and monsoon rainfall and accelerates
melting of mountain glaciers such as the Hindu Kush-Himalayan glaciers.
Q Source:Improvisation: ICSE Environmental Studies Xth

TS

99 What is the major theme running common between the Central Indian and Rajasthani
schools of painting?
A. They are deeply rooted in the Indian traditions.
B. They are highly secular in nature.
C. They are unaffected by historical developments and carry a unique style.
D. They use light colours to portray extra-terrestrial themes.
User Answer : A
Correct Answer : A
Answer Justification :

IN
SI

Justification:Unlike Mughal painting which is primarily secular, the art of painting


in Central India, Rajasthani and the Pahari region etc. is deeply rooted in the Indian
traditions, taking inspiration from Indian epics, religious texts like the Puranas, love
poems in Sanskrit and other Indian languages, Indian folk-lore and works on
musical themes.
The cults of Vaishnavism, Saivism and Sakti exercised tremendous influence on the
pictorial art of these places.
Among these the cult of Krishna was the most popular one which inspired the
patrons and artists.
The Rajasthani style of painting including that of Malwa, is marked by bold
drawing, strong and contrasting colours. The treatment of figures is flat without any
attempt to show perspective in a naturalistic manner.
Q Source:http://www.ccrtindia.gov.in/visualarts.php

(C) Insights Active Learning. | All rights reserved.

www.insightsias.com

67

TEST - 22

User Name :

chandan paswan

Total Marks

200

Mark Scored

76

IA

100 The Mount Abu InfraRed Observatory is the first major facility in India specifically
designed for ground based infrared observations of celestial objects. It is located at
Gurushikhar (highest peak of Aravali range). Why was the location specifically chosen for
the observatory?
A. It is the highest point available in Northern India.
B. Due to low moisture and higher number of cloud free nights
C. Due to better alignment with the magnetic field lines of earth
D. Temperature of the place is nearly constant throughout the year helping
infrared sensing.
User Answer : C
Correct Answer : B
Answer Justification :

TS

Justification & Learning:After a suitable astronomical site survey of several


locations, Gurushikhar, the highest peak of Aravali range in Mt Abu, Rajasthan, was
found most appropriate due to its low water vapour, more than 220 observable
nights with reasonably good seeing and the site's vicinity to a Physics Research
Laboratory.

This Infrared Observatory is situated at an altitude of 1680 meters above mean sea
level.

The observatory campus also houses an atmospheric and space science laboratory to
perform studies related to aerosols.

IN
SI

Q Source:Chapter 25: India Yearbook 2016

(C) Insights Active Learning. | All rights reserved.

www.insightsias.com

68

Vous aimerez peut-être aussi